Confusion Regarding Munkres's Definition of Basis for a Topology











up vote
5
down vote

favorite












The definition of Basis for a Topology as given in Munkres's book is as follows,




If $X$ is a set, a basis for a topology on $X$ is a collection $mathcal{B}$ of subsets of $X$ (called basis elements) such that




  • For each $x∈X$, there is at least one basis element $B$ containing $x$


  • If $x$ belongs to the intersection of two basis elements $B_1$ and $B_2$, then there is a basis
    element $B_3$ containing $x$ such that $B_3⊆B_1∩B_2$.





My question is,




In the definition though we are defining "basis for a topology", there is no mention of the topology on $X$. What role the topology on $X$ plays in the definition? Is it just a simple printing mistake or I am missing something?











share|cite|improve this question


















  • 2




    The definition only states what is necessary and sufficient for a collection $mathcal B$ to serve as base of some topology. There is no need to mention this topology. Fortunately, if $mathcal B$ has the desired properties then there is only one topology for which it can serve as base: the unions of elements of $mathcal B$ form this topology.
    – drhab
    Jul 7 '16 at 15:23












  • Many authors say "base" (which I prefer) instead of "basis". In the context of topological vector spaces we may have to consider an algebraic kind of basis, like a Hamel (vector-space) basis, or a Schauder basis, or a Hilbert-space basis, while also considering a topological $base.....$ One problem with English is that "bases" is the plural of base and also of basis.
    – DanielWainfleet
    Nov 13 at 9:57








  • 1




    @DanielWainfleet The plurals of "base" and "basis" are spelled the same but the pronunciations are different: one rhymes with "laces", the other with "Macy's".
    – bof
    Nov 16 at 6:38















up vote
5
down vote

favorite












The definition of Basis for a Topology as given in Munkres's book is as follows,




If $X$ is a set, a basis for a topology on $X$ is a collection $mathcal{B}$ of subsets of $X$ (called basis elements) such that




  • For each $x∈X$, there is at least one basis element $B$ containing $x$


  • If $x$ belongs to the intersection of two basis elements $B_1$ and $B_2$, then there is a basis
    element $B_3$ containing $x$ such that $B_3⊆B_1∩B_2$.





My question is,




In the definition though we are defining "basis for a topology", there is no mention of the topology on $X$. What role the topology on $X$ plays in the definition? Is it just a simple printing mistake or I am missing something?











share|cite|improve this question


















  • 2




    The definition only states what is necessary and sufficient for a collection $mathcal B$ to serve as base of some topology. There is no need to mention this topology. Fortunately, if $mathcal B$ has the desired properties then there is only one topology for which it can serve as base: the unions of elements of $mathcal B$ form this topology.
    – drhab
    Jul 7 '16 at 15:23












  • Many authors say "base" (which I prefer) instead of "basis". In the context of topological vector spaces we may have to consider an algebraic kind of basis, like a Hamel (vector-space) basis, or a Schauder basis, or a Hilbert-space basis, while also considering a topological $base.....$ One problem with English is that "bases" is the plural of base and also of basis.
    – DanielWainfleet
    Nov 13 at 9:57








  • 1




    @DanielWainfleet The plurals of "base" and "basis" are spelled the same but the pronunciations are different: one rhymes with "laces", the other with "Macy's".
    – bof
    Nov 16 at 6:38













up vote
5
down vote

favorite









up vote
5
down vote

favorite











The definition of Basis for a Topology as given in Munkres's book is as follows,




If $X$ is a set, a basis for a topology on $X$ is a collection $mathcal{B}$ of subsets of $X$ (called basis elements) such that




  • For each $x∈X$, there is at least one basis element $B$ containing $x$


  • If $x$ belongs to the intersection of two basis elements $B_1$ and $B_2$, then there is a basis
    element $B_3$ containing $x$ such that $B_3⊆B_1∩B_2$.





My question is,




In the definition though we are defining "basis for a topology", there is no mention of the topology on $X$. What role the topology on $X$ plays in the definition? Is it just a simple printing mistake or I am missing something?











share|cite|improve this question













The definition of Basis for a Topology as given in Munkres's book is as follows,




If $X$ is a set, a basis for a topology on $X$ is a collection $mathcal{B}$ of subsets of $X$ (called basis elements) such that




  • For each $x∈X$, there is at least one basis element $B$ containing $x$


  • If $x$ belongs to the intersection of two basis elements $B_1$ and $B_2$, then there is a basis
    element $B_3$ containing $x$ such that $B_3⊆B_1∩B_2$.





My question is,




In the definition though we are defining "basis for a topology", there is no mention of the topology on $X$. What role the topology on $X$ plays in the definition? Is it just a simple printing mistake or I am missing something?








general-topology definition






share|cite|improve this question













share|cite|improve this question











share|cite|improve this question




share|cite|improve this question










asked Jul 7 '16 at 13:49









user 170039

10.4k42463




10.4k42463








  • 2




    The definition only states what is necessary and sufficient for a collection $mathcal B$ to serve as base of some topology. There is no need to mention this topology. Fortunately, if $mathcal B$ has the desired properties then there is only one topology for which it can serve as base: the unions of elements of $mathcal B$ form this topology.
    – drhab
    Jul 7 '16 at 15:23












  • Many authors say "base" (which I prefer) instead of "basis". In the context of topological vector spaces we may have to consider an algebraic kind of basis, like a Hamel (vector-space) basis, or a Schauder basis, or a Hilbert-space basis, while also considering a topological $base.....$ One problem with English is that "bases" is the plural of base and also of basis.
    – DanielWainfleet
    Nov 13 at 9:57








  • 1




    @DanielWainfleet The plurals of "base" and "basis" are spelled the same but the pronunciations are different: one rhymes with "laces", the other with "Macy's".
    – bof
    Nov 16 at 6:38














  • 2




    The definition only states what is necessary and sufficient for a collection $mathcal B$ to serve as base of some topology. There is no need to mention this topology. Fortunately, if $mathcal B$ has the desired properties then there is only one topology for which it can serve as base: the unions of elements of $mathcal B$ form this topology.
    – drhab
    Jul 7 '16 at 15:23












  • Many authors say "base" (which I prefer) instead of "basis". In the context of topological vector spaces we may have to consider an algebraic kind of basis, like a Hamel (vector-space) basis, or a Schauder basis, or a Hilbert-space basis, while also considering a topological $base.....$ One problem with English is that "bases" is the plural of base and also of basis.
    – DanielWainfleet
    Nov 13 at 9:57








  • 1




    @DanielWainfleet The plurals of "base" and "basis" are spelled the same but the pronunciations are different: one rhymes with "laces", the other with "Macy's".
    – bof
    Nov 16 at 6:38








2




2




The definition only states what is necessary and sufficient for a collection $mathcal B$ to serve as base of some topology. There is no need to mention this topology. Fortunately, if $mathcal B$ has the desired properties then there is only one topology for which it can serve as base: the unions of elements of $mathcal B$ form this topology.
– drhab
Jul 7 '16 at 15:23






The definition only states what is necessary and sufficient for a collection $mathcal B$ to serve as base of some topology. There is no need to mention this topology. Fortunately, if $mathcal B$ has the desired properties then there is only one topology for which it can serve as base: the unions of elements of $mathcal B$ form this topology.
– drhab
Jul 7 '16 at 15:23














Many authors say "base" (which I prefer) instead of "basis". In the context of topological vector spaces we may have to consider an algebraic kind of basis, like a Hamel (vector-space) basis, or a Schauder basis, or a Hilbert-space basis, while also considering a topological $base.....$ One problem with English is that "bases" is the plural of base and also of basis.
– DanielWainfleet
Nov 13 at 9:57






Many authors say "base" (which I prefer) instead of "basis". In the context of topological vector spaces we may have to consider an algebraic kind of basis, like a Hamel (vector-space) basis, or a Schauder basis, or a Hilbert-space basis, while also considering a topological $base.....$ One problem with English is that "bases" is the plural of base and also of basis.
– DanielWainfleet
Nov 13 at 9:57






1




1




@DanielWainfleet The plurals of "base" and "basis" are spelled the same but the pronunciations are different: one rhymes with "laces", the other with "Macy's".
– bof
Nov 16 at 6:38




@DanielWainfleet The plurals of "base" and "basis" are spelled the same but the pronunciations are different: one rhymes with "laces", the other with "Macy's".
– bof
Nov 16 at 6:38










3 Answers
3






active

oldest

votes

















up vote
6
down vote



accepted










This is a completely revised version of my previous answer, which was not correct. The existence of a major error in my previous answer was recently pointed out to me by Merk Zockerborg.



The main result relevant to the question asked by user 170039 is Corollary 2 below.



Contents




  1. Basis for a topological space $(X,tau).$


  2. Given ${mathcal B} subseteq {cal P}(X),$ find a topology ${tau}_{mathcal B}$ on $X$ such that $mathcal B$ is a basis for $(X,tau_{mathcal B}).$


  3. Comparing $tau$ and $tau_{mathcal B}$ when $(X,tau)$ is a topological space and ${mathcal B} subseteq {cal P}(X)$ is such that $(X,tau_{mathcal B})$ is a topological space.



Notation: $X$ is a set and ${cal P}(X)$ is the collection of all subsets of $X.$ The (sometimes subscripted) symbols $tau$ and $mathcal B$ will denote subsets of ${cal P}(X).$ Most of the time (but not always) $tau$ will be a topology on $X$ and $mathcal B$ will be a basis for a topology on $X.$



In the spirit of making this an expository overview of some introductory ideas that arise when introducing the notion of a basis for a topological space, I am not including proofs of the stated theorems. However, those actively studying this topic in topology may find it useful to provide proofs.



1. Basis for a topological space $(X,tau).$




Definition: Let $(X,tau)$ be a topological space. A basis for $(X,tau)$ is a collection $mathcal B$ of subsets of $X$ such that:



(a) $;;mathcal B subseteq tau$
(b) $;$ For each $x in X,$ and for each $U in tau$ such that $x in U,$ there exists $V in mathcal B$ such that $x in V$ and $V subseteq U.$




Note: The term "base" is more commonly used than "basis", but I'll use "basis" in agreement with the usage by user 170039.



Roughly speaking, if we consider (open) neighborhoods of $x$ as "measures of being close to $x$", where being closer to $x$ is described by the use of a smaller (in the subset sense) neighborhood of $x,$ then a basis for $tau$ is a subcollection of open sets that is sufficient to describe being arbitrarily close to any given point.



Note that this notion of being close to $x$ has more structure to it than a notion entirely based on the subset relation, because we also have the property that any finite number of "close to $x$ conditions" can be replaced by a single "close to $x$ condition", due to the finite intersection property of open sets. More specifically, being simultaneously $U_1$-close to $x$ and $U_2$-close to $x$ can be replaced by being $(U_1 cap U_2)$-close to $x$ in the case of $tau,$ and by being $V$-close to $x$ for some $mathcal B$-element $V$ such that $V subseteq U overset{text{def}}{=} U_1 cap U_2$ in the case of $mathcal B.$ For example, suppose $X = {x,y,z}$ and $tau = {;{x,y}, ; {y,z};}.$ Note that $tau$ is not a topology on $X.$ Nonetheless, we can still talk about being close to elements of $X.$ For example, there is the notion of being "${x,y}$-close to $y$" and there is the notion of being ${y,z}$-close to $y.$ However, note that there is no single closeness condition that implies each of these conditions, because the only subset of $X$ containing $y$ that is a subset of ${x,y}$ and a subset of ${y,z}$ is ${y},$ and ${y} not in tau.$



A topology can have more than one basis. For example, in the case of the real numbers with its usual topology, the collection of all open intervals of finite length is a basis and the collection of all open intervals of finite length with rational endpoints is a basis.



Question: [This occurred to me when I was writing these notes, and I thought others here might be interested.] How many bases exist for the usual topology of the real numbers? The answer is $2^{c}.$ Since each such basis is a collection of open sets, and there are $2^c$ many collections of open sets of $mathbb R$ (because there are $c$ many open sets of ${mathbb R}),$ it follows that there are at most $2^c$ many bases for the usual topology of the real numbers. The following shows that there are at least $2^c$ many bases for the usual topology of the real numbers. Let $D_1$ and $D_2$ be disjoint subsets of ${mathbb R}$ such that $D_1$ has cardinality $c$ and $D_2$ is dense-in-${mathbb R}$ (for example, $D_1$ could be the set of irrational numbers between $0$ and $1,$ and $D_2$ could be the set of rational numbers), and consider the collection of open intervals each of whose endpoint(s) belongs to $D_1 cup D_{2}.$ This collection of open intervals is a basis for ${mathbb R}.$ Moreover, if we remove from this collection any set consisting only of open intervals each of whose endpoint(s) belongs to $D_{1},$ then what remains will also be a basis, and there are $2^c$ many ways to remove such sets of intervals from the original collection of open intervals (because there are $2^c$ many subsets of the collection of open intervals each of whose endpoint(s) belongs to $D_{1}).$




Theorem 1: Let $(X,tau)$ be a topological space and let $mathcal B$ be a collection of subsets of $X$ such that:



(a) $;;mathcal B subseteq tau$
(b) $;$ Every element of $tau$ is the union of some subcollection of elements in $mathcal B.$



Then $mathcal B$ is a basis for $(X,tau).$




Regarding (b) above, we are using the convention that an empty union of sets is possible, and that an empty union of sets is equal to the empty set.




Theorem 2: Let $(X,tau)$ be a topological space and let $mathcal B$ be a basis for $(X,tau).$ Then every element of $tau$ is the union of some subcollection of elements in $mathcal B.$




Theorems 1 and 2 together imply that we could have defined "$mathcal B$ is a basis for $(X,tau)$" by replacing (b) in our earlier definition with (b) in Theorem 1. This alternate way of defining a basis for $(X,tau)$ is used in some books.



Note that (b) in Theorem 1 provides a way to generate all elements of $tau$ from the elements in $mathcal B$ — take unions. This is analogous to having a way to generate all elements in a vector space from a (vector space) basis — take linear combinations.



2. Given ${mathcal B} subseteq {cal P}(X),$ find a topology ${tau}_{mathcal B}$ on $X$ such that $mathcal B$ is a basis for $(X,tau_{mathcal B}).$



Suppose that no topology on the set $X$ has been provided. Can we obtain a topology ${tau}_{mathcal B}$ on $X$ by picking some collection $mathcal B subseteq {cal P}(X)$ and letting ${tau}_{mathcal B}$ be the collection of all possible unions of elements in $mathcal B$?




Example 1: Let $X = {x,y}$ and let $mathcal B = {; {x};}.$ Then the collection of all possible unions of elements in $mathcal B$ gives ${tau}_{mathcal B} = {; emptyset,;{x};},$ but ${tau}_{mathcal B}$ is not a topology on $X,$ because $X notin {tau}_{mathcal B}.$ (A simpler example is to use $mathcal B = emptyset,$ but I wanted to give a less trivial example.)




Perhaps we can fix the problem that arises in Example 1 by assuming $cup {mathcal B} = X.$ Note that $cup {mathcal B} = X$ is equivalent to the first bullet assumption in user 170039’s statement of the definition of "basis for a topology" from Munkres's book. Example 2 shows that having $cup {mathcal B} = X$ is not sufficient for ${tau}_{mathcal B}$ to be a topology on $X.$




Example 2: Let $X = {x,y,z}$ and let ${mathcal B} = {;{x,y},;{y,z};}.$ Then $cup {mathcal B} = X.$ However, the collection of all possible unions of elements in $mathcal B$ gives ${tau}_{mathcal B} = {;emptyset,; {x,y},;{y,z},; X;},$ and ${tau}_{mathcal B}$ is not a topology because ${tau}_{mathcal B}$ is not closed under finite intersections — note that ${x,y} cap {y,z} = {y}$ and ${y} notin {tau}_{mathcal B}.$




It turns out that fixing both problems, namely the problem that Example 1 illustrates and the problem that Example 2 illustrates, is sufficient for ${tau}_{mathcal B}$ to be a topology on $X.$




Theorem 3: Let $X$ be a set and let $mathcal B$ be a collection of subsets of $X$ such that:



(a) $;; cup {mathcal B} = X$
(b) $;$ For each positive integer $n,$ if $B_1 in mathcal B$ and $B_2 in mathcal B$ and $cdots$ and $B_n in {mathcal B},$ then $B_1 cap B_2 cap cdots cap B_n$ can be written as the union of (possibly infinitely many) elements in ${mathcal B}.$



If we let ${tau}_{mathcal B}$ be the collection of all possible unions of elements in $mathcal B,$ then $(X,{tau}_{mathcal B})$ is a topological space and $mathcal B$ is a basis for $(X,{tau}_{mathcal B}).$




In some books Theorem 3 is buried in results about a subbasis for a topological space. Explicit statements of it can be found in Theorem 3.50 on p. 26 of Introduction to General Topology by Helen Frances Cullen (1968) and in 2.2 Base Characterization on p. 153 of An Introduction to Topology and Homotopy by Allan John Sieradski (1992). On the off-chance that anyone looks at Cullen's book, note that in her book she assumes an empty intersection gives the underlying universal set (see top of p. 416). Since this is not a standard assumption, I've avoided this assumption by including (a).



Theorem 4 is the more commonly stated condition for a collection of subsets of $X$ to generate a topology on $X,$ where the method of generation is by using the collection of all possible unions of elements in that collection of subsets. At the risk of stating the obvious, if $mathcal B$ satisfies the assumptions in Theorem 3 and $mathcal B$ also satisfies the assumptions in Theorem 4 (in fact, one can show that satisfying the assumptions in either theorem implies satisfying the assumptions in the other theorem), then the topology obtained using Theorem 3 is the same as the topology obtained using Theorem 4. This follows from the fact that in each theorem the same procedure is used to obtain the topology, namely the topology is the collection of all possible unions of elements in ${mathcal B}.$




Theorem 4: Let $X$ be a set and let $mathcal B$ be a collection of subsets of $X$ such that:



(a) $;; cup {mathcal B} = X$
(b) $;$ If $U in mathcal B$ and $V in mathcal B$ and $x in U cap V,$ then there exists $W in mathcal B$ such that $x in W$ and $W subseteq U cap V.$



If we let ${tau}_{mathcal B}$ be the collection of all possible unions of elements in $mathcal B,$ then $(X,{tau}_{mathcal B})$ is a topological space and $mathcal B$ is a basis for $(X,{tau}_{mathcal B}).$




One the things that (b) in Theorem 4 does is to incorporate our earlier observation (long paragraph a little below the definition of "basis") that finitely many "close to $x$" conditions can be replaced by a single "close to $x$" condition. Note that in Example 2, ${tau}_{mathcal B}$ has the property that "${x,y}$-close to $y$ and ${y,z}$-close to $y$" cannot be equaled or strengthened by any single ${tau}_{mathcal B}$-closeness notion.



In our earlier vector space analogy, Theorems 3 and 4 are somewhat akin to finding conditions under which a set of vectors is a linearly independent set, then defining the linear span of those vectors, and finally observing that the linear span of those vectors forms a vector space.



3. Comparing $tau$ and $tau_{mathcal B}$ when $(X,tau)$ is a topological space and ${mathcal B} subseteq {cal P}(X)$ is such that $(X,tau_{mathcal B})$ is a topological space.



Let $(X,tau)$ be a topological space and let $mathcal B$ be a collection of subsets of $X$ satisfying the assumptions in either Theorem 3 or Theorem 4 (or both). In this situation we have two topological spaces to contend with, $(X,tau)$ and $(X,,tau_{mathcal B}).$ In general, we have $tau neq tau_{mathcal B}.$ Indeed, there is no reason why we would expect there to be any relation between $tau$ and $tau_{mathcal B},$ aside from the fact that each is a collection of subsets of $X$ that satisfies the axioms for a topology on $X.$




Example 3: Suppose ${mathcal B} not subseteq {tau}.$ Then it is easy to see that $tau neq tau_{mathcal B}.$ (Proof: Let $B in mathcal B$ such that $B not in {tau}.$ Then $B$ belongs to $tau_{mathcal B},$ because we always have ${mathcal B} subseteq tau_{mathcal B},$ and hence $B$ does not belong to ${tau}.$ Therefore, $tau neq tau_{mathcal B}.)$




Example 4, due to Merk Zockerborg, shows that ${mathcal B} subseteq {tau}$ is not sufficient to conclude that $tau = tau_{mathcal B}.$




Example 4: Let $X = {x,y}$ and $tau = {;emptyset,;{x},;{y},;{x,y};}$ and ${mathcal B} = {;{x},;{x,y};}.$ Then ${mathcal B} subseteq {tau}$ and $tau_{mathcal B} = {;emptyset,;{x},;{x,y};}.$ Therefore, $tau neq tau_{mathcal B}.$




Corollary 2 below gives one possible assumption that, along with the assumption ${mathcal B} subseteq {tau},$ is sufficient to conclude that $tau = tau_{mathcal B}.$




Theorem 5: Let $X$ be a set and let ${mathcal B}_1,$ ${mathcal B}_2$ each be collections of subsets of $X$ that satisfy the assumptions in either Theorem 3 or Theorem 4. Then ${tau}_{{mathcal B}_1} subseteq {tau}_{{mathcal B}_2}$ if and only if the following assertion holds:



For each $B_1 in {mathcal B}_1$ and for each $x in B_{1},$ there exists $B_2 in {mathcal B}_2$ such that $x in B_2$ and $B_2 subseteq B_{1}.$




$;$




Corollary 1: Let $X$ be a set and let ${mathcal B}_1,$ ${mathcal B}_2$ each be collections of subsets of $X$ that satisfy the assumptions in either Theorem 3 or Theorem 4. Then ${tau}_{{mathcal B}_1} = {tau}_{{mathcal B}_2}$ if and only if both of the following assertions hold:



For each $B_1 in {mathcal B}_1$ and for each $x in B_{1},$ there exists $B_2 in {mathcal B}_2$ such that $x in B_2$ and $B_2 subseteq B_{1}.$

For each $B_2 in {mathcal B}_2$ and for each $x in B_{2},$ there exists $B_1 in {mathcal B}_1$ such that $x in B_1$ and $B_1 subseteq B_{2}.$




$;$




Corollary 2: Let $(X,tau)$ be a topological space and let $mathcal B$ be a collection of subsets of $X$ that satisfies the assumptions in either Theorem 3 or Theorem 4 such that:



(a) $;; {mathcal B} subseteq {tau}$
(b) $;$ For each $U in tau$ and for each $x in U,$ there exists $B in mathcal B$ such that $x in B$ and $B subseteq U.$



Then $tau = tau_{mathcal B}.$




Proof of Corollary 2: From ${mathcal B} subseteq {tau}$ and the fact that $tau$ is closed under arbitrary unions, it follows that $tau_{mathcal B} subseteq {tau}.$ Applying Theorem 5 with ${mathcal B}_1 = tau$ and ${mathcal B}_2 = {mathcal B},$ and observing that ${tau}_{{mathcal B}_1} = {tau}_{tau} = tau$ (because $tau$ is closed under arbitrary unions), we get $tau subseteq tau_{mathcal B}.$ (The notation is slightly confusing, since in ${tau}_{tau}$ the subscripted $tau$ is a topology on $X$ and the other $tau$ is part of the notation that symbolizes the result of carrying out a certain operation on subscripted set.)






share|cite|improve this answer























  • You are right. That's what we have discussed here and here.
    – user 170039
    Jul 7 '16 at 14:57






  • 1




    sorry to dig up a long inactive post, but I think that the claim $mathcal B subseteq {tau} implies tau = tau_{mathcal B}$ is incorrect? For instance consider the Sierpinski topology on $X={0,1}$, which has $tau_{mathcal B} = { emptyset, {1}, {0, 1} }$ generated by basis sets ${1}$ and ${0,1}$. Consider the discrete topology $tau = { emptyset, {0}, {1}, {0,1} }$. Clearly $mathcal B subset tau$ but $tau_{mathcal B} neq tau$.
    – Merk Zockerborg
    Nov 8 at 14:26








  • 1




    @Merk Zockerborg: I'll look at this tomorrow when I have more time, but for now I think you are correct and more is needed to conclude the two topologies are equal. I think I know what's missing, but I want to preserve the simplicity of my answer (which appears to have been simplified too much!), and to carefully rewrite my answer to incorporate what's needed, it's best that I do it in the morning hours when I'm fresher.
    – Dave L. Renfro
    Nov 8 at 21:17












  • @Merk Zockerborg: It wasn't until yesterday (day after "tomorrow" in my earlier comment) that I began looking at this, and it appears that a very extensive revision is needed. In fact, I'm completely starting all over again. However, given the kinds of confusion this topic tends to generate, and the typically quick way it's taken care of in topology texts, I think it would be useful to have in Stack Exchange a discussion that fleshes out all the various nuances in a careful way. Not sure I'll have it done today, but I'm working some more on it right now.
    – Dave L. Renfro
    Nov 11 at 18:39












  • @Merk Zockerborg: I've just posted a lengthy replacement essay. I mostly finished yesterday (except for two corollaries at the very end), but I wanted to go over everything again, which I did this morning. Ideally, I'd like to give it another "read through and tinkering" tomorrow morning, because there are so many details I could have slipped up on or typos made, but I have some extensive contract work I need to get busy with this week, so I'm posting what I have at this time so I can (hopefully) put it behind me and get to work on the other stuff.
    – Dave L. Renfro
    Nov 12 at 15:07




















up vote
2
down vote













You don't need a topology to define a base for a topology, but every basis generates a topology, and you can go about finding bases for existing topologies.



The topology generated by such a basis is determined in the following way:



A subset $mathcal O$ of $X$ is open if for all $xin mathcal O$, there is an element $mathcal B$ of the basis such that $xin mathcal Bsubseteq mathcal O$.



Expressed another way, you can say that the open subsets are the ones which are unions of basis elements.



Why use bases? Well, just like in liner algebra, it's a way to compress information about all open sets into a smaller collection that recovers everything else. For example, the open balls of rational radius around points in $mathbb R^2$ form a countable basis of the ordinary topology on $Bbb R^2$. This collection is not closed under intersection or union, and the full topology on $mathbb R$ contains uncountably many open sets.






share|cite|improve this answer























  • How does this answer my question?
    – user 170039
    Jul 7 '16 at 13:56










  • I guess I should have said (which I have now inserted) that it is not necessary to have a topology before defining a basis. You can define a basis then generate a topology, or else you can find a topology for an existing basis.
    – rschwieb
    Jul 7 '16 at 13:59










  • So by "basis for a topology.." he means "basis for some topology (yet to be determined)", am I right?
    – user 170039
    Jul 7 '16 at 14:04










  • @user170039 It can be: if you define a basis according to these axioms, you can generate a topology. But sometimes it goes the other way where you have a topology and you judiciously choose a basis for it to simplify your proofs.
    – rschwieb
    Jul 7 '16 at 14:08












  • The problem is that suppose we have a topology on $X$, say $mathcal{T}$. Then from the definition, it doesn't follow that a basis for $mathcal{T}$, say $mathcal{B}$ is a subset of $mathcal{T}$. To be precise, let $X=mathbb{R}$ with the standard topology. Then observe that both the set $mathcal{B}_{1}:={(a,b):a,binmathbb{R}}$ and $mathcal{B}_{1}:={[a,b]:a,binmathbb{R}}$ can be considered as a basis for the usual topology on $mathbb{R}$ according to the definition.
    – user 170039
    Jul 7 '16 at 14:14


















up vote
2
down vote













A bit further in the book he will discuss how such a basis on a set generates a basis for a topology on that set by taking unions of basis elements.





Conversely, if you start with a space $X$ which is already topologized, then a collection $mathcal{B}$ of open subsets such that any open subset of $X$ can be written as a union of elements of $mathcal{B}$ is a basis for the set $X$ in the sense of the definition above. (And, via this basis, we recover the topology on $X$.)



Proof. We check the first condition. Let $x in X$. Since $X$ is open as a subset of itself, it may be written as a union of basis elements, $X = bigcup_mathcal{B} B$. Therefore by the definition of union, $x$ belongs to some $B in mathcal{B}$.



Now for the second condition. If $x$ belongs to the two open sets $B_1,B_2 in mathcal{B}$, it must also belong to the open set $B_1 cap B_2$. Writing this set as a union of basis elements, $B_1 cap B_2 = bigcup_mathcal{B} B$, we see again that by the definition of union there must exist some basis element $B_3 subset B_1 cap B_2$ containing $x$.






share|cite|improve this answer























  • How does this answer my question?
    – user 170039
    Jul 7 '16 at 13:56










  • @user170039 Is your question "What role does the topology on $X$ play in the definition?" If so, my answer is: your question is meaningless because $X$ isn't endowed with a topology.
    – Alex Provost
    Jul 7 '16 at 13:58












  • @user170039 The point is that you start with a set $X$ (with no topology) and use the basis to construct a topology on $X$.
    – Alex Provost
    Jul 7 '16 at 13:59












  • How does the converse part follows from the definition? Suppose $X$ is a set endowed with topology $mathcal{T}$. In the definition it is only mentioned that, "a basis for a topology on $X$ is a collection $mathcal{B}$ of subsets of $X$..", not as you wrote, "a collection of open subsets".
    – user 170039
    Jul 7 '16 at 14:10










  • @user170039 I wrote a proof of the converse. A collection of open subsets is, in particular, a collection of subsets...
    – Alex Provost
    Jul 7 '16 at 14:24











Your Answer





StackExchange.ifUsing("editor", function () {
return StackExchange.using("mathjaxEditing", function () {
StackExchange.MarkdownEditor.creationCallbacks.add(function (editor, postfix) {
StackExchange.mathjaxEditing.prepareWmdForMathJax(editor, postfix, [["$", "$"], ["\\(","\\)"]]);
});
});
}, "mathjax-editing");

StackExchange.ready(function() {
var channelOptions = {
tags: "".split(" "),
id: "69"
};
initTagRenderer("".split(" "), "".split(" "), channelOptions);

StackExchange.using("externalEditor", function() {
// Have to fire editor after snippets, if snippets enabled
if (StackExchange.settings.snippets.snippetsEnabled) {
StackExchange.using("snippets", function() {
createEditor();
});
}
else {
createEditor();
}
});

function createEditor() {
StackExchange.prepareEditor({
heartbeatType: 'answer',
convertImagesToLinks: true,
noModals: true,
showLowRepImageUploadWarning: true,
reputationToPostImages: 10,
bindNavPrevention: true,
postfix: "",
imageUploader: {
brandingHtml: "Powered by u003ca class="icon-imgur-white" href="https://imgur.com/"u003eu003c/au003e",
contentPolicyHtml: "User contributions licensed under u003ca href="https://creativecommons.org/licenses/by-sa/3.0/"u003ecc by-sa 3.0 with attribution requiredu003c/au003e u003ca href="https://stackoverflow.com/legal/content-policy"u003e(content policy)u003c/au003e",
allowUrls: true
},
noCode: true, onDemand: true,
discardSelector: ".discard-answer"
,immediatelyShowMarkdownHelp:true
});


}
});














 

draft saved


draft discarded


















StackExchange.ready(
function () {
StackExchange.openid.initPostLogin('.new-post-login', 'https%3a%2f%2fmath.stackexchange.com%2fquestions%2f1852072%2fconfusion-regarding-munkress-definition-of-basis-for-a-topology%23new-answer', 'question_page');
}
);

Post as a guest















Required, but never shown

























3 Answers
3






active

oldest

votes








3 Answers
3






active

oldest

votes









active

oldest

votes






active

oldest

votes








up vote
6
down vote



accepted










This is a completely revised version of my previous answer, which was not correct. The existence of a major error in my previous answer was recently pointed out to me by Merk Zockerborg.



The main result relevant to the question asked by user 170039 is Corollary 2 below.



Contents




  1. Basis for a topological space $(X,tau).$


  2. Given ${mathcal B} subseteq {cal P}(X),$ find a topology ${tau}_{mathcal B}$ on $X$ such that $mathcal B$ is a basis for $(X,tau_{mathcal B}).$


  3. Comparing $tau$ and $tau_{mathcal B}$ when $(X,tau)$ is a topological space and ${mathcal B} subseteq {cal P}(X)$ is such that $(X,tau_{mathcal B})$ is a topological space.



Notation: $X$ is a set and ${cal P}(X)$ is the collection of all subsets of $X.$ The (sometimes subscripted) symbols $tau$ and $mathcal B$ will denote subsets of ${cal P}(X).$ Most of the time (but not always) $tau$ will be a topology on $X$ and $mathcal B$ will be a basis for a topology on $X.$



In the spirit of making this an expository overview of some introductory ideas that arise when introducing the notion of a basis for a topological space, I am not including proofs of the stated theorems. However, those actively studying this topic in topology may find it useful to provide proofs.



1. Basis for a topological space $(X,tau).$




Definition: Let $(X,tau)$ be a topological space. A basis for $(X,tau)$ is a collection $mathcal B$ of subsets of $X$ such that:



(a) $;;mathcal B subseteq tau$
(b) $;$ For each $x in X,$ and for each $U in tau$ such that $x in U,$ there exists $V in mathcal B$ such that $x in V$ and $V subseteq U.$




Note: The term "base" is more commonly used than "basis", but I'll use "basis" in agreement with the usage by user 170039.



Roughly speaking, if we consider (open) neighborhoods of $x$ as "measures of being close to $x$", where being closer to $x$ is described by the use of a smaller (in the subset sense) neighborhood of $x,$ then a basis for $tau$ is a subcollection of open sets that is sufficient to describe being arbitrarily close to any given point.



Note that this notion of being close to $x$ has more structure to it than a notion entirely based on the subset relation, because we also have the property that any finite number of "close to $x$ conditions" can be replaced by a single "close to $x$ condition", due to the finite intersection property of open sets. More specifically, being simultaneously $U_1$-close to $x$ and $U_2$-close to $x$ can be replaced by being $(U_1 cap U_2)$-close to $x$ in the case of $tau,$ and by being $V$-close to $x$ for some $mathcal B$-element $V$ such that $V subseteq U overset{text{def}}{=} U_1 cap U_2$ in the case of $mathcal B.$ For example, suppose $X = {x,y,z}$ and $tau = {;{x,y}, ; {y,z};}.$ Note that $tau$ is not a topology on $X.$ Nonetheless, we can still talk about being close to elements of $X.$ For example, there is the notion of being "${x,y}$-close to $y$" and there is the notion of being ${y,z}$-close to $y.$ However, note that there is no single closeness condition that implies each of these conditions, because the only subset of $X$ containing $y$ that is a subset of ${x,y}$ and a subset of ${y,z}$ is ${y},$ and ${y} not in tau.$



A topology can have more than one basis. For example, in the case of the real numbers with its usual topology, the collection of all open intervals of finite length is a basis and the collection of all open intervals of finite length with rational endpoints is a basis.



Question: [This occurred to me when I was writing these notes, and I thought others here might be interested.] How many bases exist for the usual topology of the real numbers? The answer is $2^{c}.$ Since each such basis is a collection of open sets, and there are $2^c$ many collections of open sets of $mathbb R$ (because there are $c$ many open sets of ${mathbb R}),$ it follows that there are at most $2^c$ many bases for the usual topology of the real numbers. The following shows that there are at least $2^c$ many bases for the usual topology of the real numbers. Let $D_1$ and $D_2$ be disjoint subsets of ${mathbb R}$ such that $D_1$ has cardinality $c$ and $D_2$ is dense-in-${mathbb R}$ (for example, $D_1$ could be the set of irrational numbers between $0$ and $1,$ and $D_2$ could be the set of rational numbers), and consider the collection of open intervals each of whose endpoint(s) belongs to $D_1 cup D_{2}.$ This collection of open intervals is a basis for ${mathbb R}.$ Moreover, if we remove from this collection any set consisting only of open intervals each of whose endpoint(s) belongs to $D_{1},$ then what remains will also be a basis, and there are $2^c$ many ways to remove such sets of intervals from the original collection of open intervals (because there are $2^c$ many subsets of the collection of open intervals each of whose endpoint(s) belongs to $D_{1}).$




Theorem 1: Let $(X,tau)$ be a topological space and let $mathcal B$ be a collection of subsets of $X$ such that:



(a) $;;mathcal B subseteq tau$
(b) $;$ Every element of $tau$ is the union of some subcollection of elements in $mathcal B.$



Then $mathcal B$ is a basis for $(X,tau).$




Regarding (b) above, we are using the convention that an empty union of sets is possible, and that an empty union of sets is equal to the empty set.




Theorem 2: Let $(X,tau)$ be a topological space and let $mathcal B$ be a basis for $(X,tau).$ Then every element of $tau$ is the union of some subcollection of elements in $mathcal B.$




Theorems 1 and 2 together imply that we could have defined "$mathcal B$ is a basis for $(X,tau)$" by replacing (b) in our earlier definition with (b) in Theorem 1. This alternate way of defining a basis for $(X,tau)$ is used in some books.



Note that (b) in Theorem 1 provides a way to generate all elements of $tau$ from the elements in $mathcal B$ — take unions. This is analogous to having a way to generate all elements in a vector space from a (vector space) basis — take linear combinations.



2. Given ${mathcal B} subseteq {cal P}(X),$ find a topology ${tau}_{mathcal B}$ on $X$ such that $mathcal B$ is a basis for $(X,tau_{mathcal B}).$



Suppose that no topology on the set $X$ has been provided. Can we obtain a topology ${tau}_{mathcal B}$ on $X$ by picking some collection $mathcal B subseteq {cal P}(X)$ and letting ${tau}_{mathcal B}$ be the collection of all possible unions of elements in $mathcal B$?




Example 1: Let $X = {x,y}$ and let $mathcal B = {; {x};}.$ Then the collection of all possible unions of elements in $mathcal B$ gives ${tau}_{mathcal B} = {; emptyset,;{x};},$ but ${tau}_{mathcal B}$ is not a topology on $X,$ because $X notin {tau}_{mathcal B}.$ (A simpler example is to use $mathcal B = emptyset,$ but I wanted to give a less trivial example.)




Perhaps we can fix the problem that arises in Example 1 by assuming $cup {mathcal B} = X.$ Note that $cup {mathcal B} = X$ is equivalent to the first bullet assumption in user 170039’s statement of the definition of "basis for a topology" from Munkres's book. Example 2 shows that having $cup {mathcal B} = X$ is not sufficient for ${tau}_{mathcal B}$ to be a topology on $X.$




Example 2: Let $X = {x,y,z}$ and let ${mathcal B} = {;{x,y},;{y,z};}.$ Then $cup {mathcal B} = X.$ However, the collection of all possible unions of elements in $mathcal B$ gives ${tau}_{mathcal B} = {;emptyset,; {x,y},;{y,z},; X;},$ and ${tau}_{mathcal B}$ is not a topology because ${tau}_{mathcal B}$ is not closed under finite intersections — note that ${x,y} cap {y,z} = {y}$ and ${y} notin {tau}_{mathcal B}.$




It turns out that fixing both problems, namely the problem that Example 1 illustrates and the problem that Example 2 illustrates, is sufficient for ${tau}_{mathcal B}$ to be a topology on $X.$




Theorem 3: Let $X$ be a set and let $mathcal B$ be a collection of subsets of $X$ such that:



(a) $;; cup {mathcal B} = X$
(b) $;$ For each positive integer $n,$ if $B_1 in mathcal B$ and $B_2 in mathcal B$ and $cdots$ and $B_n in {mathcal B},$ then $B_1 cap B_2 cap cdots cap B_n$ can be written as the union of (possibly infinitely many) elements in ${mathcal B}.$



If we let ${tau}_{mathcal B}$ be the collection of all possible unions of elements in $mathcal B,$ then $(X,{tau}_{mathcal B})$ is a topological space and $mathcal B$ is a basis for $(X,{tau}_{mathcal B}).$




In some books Theorem 3 is buried in results about a subbasis for a topological space. Explicit statements of it can be found in Theorem 3.50 on p. 26 of Introduction to General Topology by Helen Frances Cullen (1968) and in 2.2 Base Characterization on p. 153 of An Introduction to Topology and Homotopy by Allan John Sieradski (1992). On the off-chance that anyone looks at Cullen's book, note that in her book she assumes an empty intersection gives the underlying universal set (see top of p. 416). Since this is not a standard assumption, I've avoided this assumption by including (a).



Theorem 4 is the more commonly stated condition for a collection of subsets of $X$ to generate a topology on $X,$ where the method of generation is by using the collection of all possible unions of elements in that collection of subsets. At the risk of stating the obvious, if $mathcal B$ satisfies the assumptions in Theorem 3 and $mathcal B$ also satisfies the assumptions in Theorem 4 (in fact, one can show that satisfying the assumptions in either theorem implies satisfying the assumptions in the other theorem), then the topology obtained using Theorem 3 is the same as the topology obtained using Theorem 4. This follows from the fact that in each theorem the same procedure is used to obtain the topology, namely the topology is the collection of all possible unions of elements in ${mathcal B}.$




Theorem 4: Let $X$ be a set and let $mathcal B$ be a collection of subsets of $X$ such that:



(a) $;; cup {mathcal B} = X$
(b) $;$ If $U in mathcal B$ and $V in mathcal B$ and $x in U cap V,$ then there exists $W in mathcal B$ such that $x in W$ and $W subseteq U cap V.$



If we let ${tau}_{mathcal B}$ be the collection of all possible unions of elements in $mathcal B,$ then $(X,{tau}_{mathcal B})$ is a topological space and $mathcal B$ is a basis for $(X,{tau}_{mathcal B}).$




One the things that (b) in Theorem 4 does is to incorporate our earlier observation (long paragraph a little below the definition of "basis") that finitely many "close to $x$" conditions can be replaced by a single "close to $x$" condition. Note that in Example 2, ${tau}_{mathcal B}$ has the property that "${x,y}$-close to $y$ and ${y,z}$-close to $y$" cannot be equaled or strengthened by any single ${tau}_{mathcal B}$-closeness notion.



In our earlier vector space analogy, Theorems 3 and 4 are somewhat akin to finding conditions under which a set of vectors is a linearly independent set, then defining the linear span of those vectors, and finally observing that the linear span of those vectors forms a vector space.



3. Comparing $tau$ and $tau_{mathcal B}$ when $(X,tau)$ is a topological space and ${mathcal B} subseteq {cal P}(X)$ is such that $(X,tau_{mathcal B})$ is a topological space.



Let $(X,tau)$ be a topological space and let $mathcal B$ be a collection of subsets of $X$ satisfying the assumptions in either Theorem 3 or Theorem 4 (or both). In this situation we have two topological spaces to contend with, $(X,tau)$ and $(X,,tau_{mathcal B}).$ In general, we have $tau neq tau_{mathcal B}.$ Indeed, there is no reason why we would expect there to be any relation between $tau$ and $tau_{mathcal B},$ aside from the fact that each is a collection of subsets of $X$ that satisfies the axioms for a topology on $X.$




Example 3: Suppose ${mathcal B} not subseteq {tau}.$ Then it is easy to see that $tau neq tau_{mathcal B}.$ (Proof: Let $B in mathcal B$ such that $B not in {tau}.$ Then $B$ belongs to $tau_{mathcal B},$ because we always have ${mathcal B} subseteq tau_{mathcal B},$ and hence $B$ does not belong to ${tau}.$ Therefore, $tau neq tau_{mathcal B}.)$




Example 4, due to Merk Zockerborg, shows that ${mathcal B} subseteq {tau}$ is not sufficient to conclude that $tau = tau_{mathcal B}.$




Example 4: Let $X = {x,y}$ and $tau = {;emptyset,;{x},;{y},;{x,y};}$ and ${mathcal B} = {;{x},;{x,y};}.$ Then ${mathcal B} subseteq {tau}$ and $tau_{mathcal B} = {;emptyset,;{x},;{x,y};}.$ Therefore, $tau neq tau_{mathcal B}.$




Corollary 2 below gives one possible assumption that, along with the assumption ${mathcal B} subseteq {tau},$ is sufficient to conclude that $tau = tau_{mathcal B}.$




Theorem 5: Let $X$ be a set and let ${mathcal B}_1,$ ${mathcal B}_2$ each be collections of subsets of $X$ that satisfy the assumptions in either Theorem 3 or Theorem 4. Then ${tau}_{{mathcal B}_1} subseteq {tau}_{{mathcal B}_2}$ if and only if the following assertion holds:



For each $B_1 in {mathcal B}_1$ and for each $x in B_{1},$ there exists $B_2 in {mathcal B}_2$ such that $x in B_2$ and $B_2 subseteq B_{1}.$




$;$




Corollary 1: Let $X$ be a set and let ${mathcal B}_1,$ ${mathcal B}_2$ each be collections of subsets of $X$ that satisfy the assumptions in either Theorem 3 or Theorem 4. Then ${tau}_{{mathcal B}_1} = {tau}_{{mathcal B}_2}$ if and only if both of the following assertions hold:



For each $B_1 in {mathcal B}_1$ and for each $x in B_{1},$ there exists $B_2 in {mathcal B}_2$ such that $x in B_2$ and $B_2 subseteq B_{1}.$

For each $B_2 in {mathcal B}_2$ and for each $x in B_{2},$ there exists $B_1 in {mathcal B}_1$ such that $x in B_1$ and $B_1 subseteq B_{2}.$




$;$




Corollary 2: Let $(X,tau)$ be a topological space and let $mathcal B$ be a collection of subsets of $X$ that satisfies the assumptions in either Theorem 3 or Theorem 4 such that:



(a) $;; {mathcal B} subseteq {tau}$
(b) $;$ For each $U in tau$ and for each $x in U,$ there exists $B in mathcal B$ such that $x in B$ and $B subseteq U.$



Then $tau = tau_{mathcal B}.$




Proof of Corollary 2: From ${mathcal B} subseteq {tau}$ and the fact that $tau$ is closed under arbitrary unions, it follows that $tau_{mathcal B} subseteq {tau}.$ Applying Theorem 5 with ${mathcal B}_1 = tau$ and ${mathcal B}_2 = {mathcal B},$ and observing that ${tau}_{{mathcal B}_1} = {tau}_{tau} = tau$ (because $tau$ is closed under arbitrary unions), we get $tau subseteq tau_{mathcal B}.$ (The notation is slightly confusing, since in ${tau}_{tau}$ the subscripted $tau$ is a topology on $X$ and the other $tau$ is part of the notation that symbolizes the result of carrying out a certain operation on subscripted set.)






share|cite|improve this answer























  • You are right. That's what we have discussed here and here.
    – user 170039
    Jul 7 '16 at 14:57






  • 1




    sorry to dig up a long inactive post, but I think that the claim $mathcal B subseteq {tau} implies tau = tau_{mathcal B}$ is incorrect? For instance consider the Sierpinski topology on $X={0,1}$, which has $tau_{mathcal B} = { emptyset, {1}, {0, 1} }$ generated by basis sets ${1}$ and ${0,1}$. Consider the discrete topology $tau = { emptyset, {0}, {1}, {0,1} }$. Clearly $mathcal B subset tau$ but $tau_{mathcal B} neq tau$.
    – Merk Zockerborg
    Nov 8 at 14:26








  • 1




    @Merk Zockerborg: I'll look at this tomorrow when I have more time, but for now I think you are correct and more is needed to conclude the two topologies are equal. I think I know what's missing, but I want to preserve the simplicity of my answer (which appears to have been simplified too much!), and to carefully rewrite my answer to incorporate what's needed, it's best that I do it in the morning hours when I'm fresher.
    – Dave L. Renfro
    Nov 8 at 21:17












  • @Merk Zockerborg: It wasn't until yesterday (day after "tomorrow" in my earlier comment) that I began looking at this, and it appears that a very extensive revision is needed. In fact, I'm completely starting all over again. However, given the kinds of confusion this topic tends to generate, and the typically quick way it's taken care of in topology texts, I think it would be useful to have in Stack Exchange a discussion that fleshes out all the various nuances in a careful way. Not sure I'll have it done today, but I'm working some more on it right now.
    – Dave L. Renfro
    Nov 11 at 18:39












  • @Merk Zockerborg: I've just posted a lengthy replacement essay. I mostly finished yesterday (except for two corollaries at the very end), but I wanted to go over everything again, which I did this morning. Ideally, I'd like to give it another "read through and tinkering" tomorrow morning, because there are so many details I could have slipped up on or typos made, but I have some extensive contract work I need to get busy with this week, so I'm posting what I have at this time so I can (hopefully) put it behind me and get to work on the other stuff.
    – Dave L. Renfro
    Nov 12 at 15:07

















up vote
6
down vote



accepted










This is a completely revised version of my previous answer, which was not correct. The existence of a major error in my previous answer was recently pointed out to me by Merk Zockerborg.



The main result relevant to the question asked by user 170039 is Corollary 2 below.



Contents




  1. Basis for a topological space $(X,tau).$


  2. Given ${mathcal B} subseteq {cal P}(X),$ find a topology ${tau}_{mathcal B}$ on $X$ such that $mathcal B$ is a basis for $(X,tau_{mathcal B}).$


  3. Comparing $tau$ and $tau_{mathcal B}$ when $(X,tau)$ is a topological space and ${mathcal B} subseteq {cal P}(X)$ is such that $(X,tau_{mathcal B})$ is a topological space.



Notation: $X$ is a set and ${cal P}(X)$ is the collection of all subsets of $X.$ The (sometimes subscripted) symbols $tau$ and $mathcal B$ will denote subsets of ${cal P}(X).$ Most of the time (but not always) $tau$ will be a topology on $X$ and $mathcal B$ will be a basis for a topology on $X.$



In the spirit of making this an expository overview of some introductory ideas that arise when introducing the notion of a basis for a topological space, I am not including proofs of the stated theorems. However, those actively studying this topic in topology may find it useful to provide proofs.



1. Basis for a topological space $(X,tau).$




Definition: Let $(X,tau)$ be a topological space. A basis for $(X,tau)$ is a collection $mathcal B$ of subsets of $X$ such that:



(a) $;;mathcal B subseteq tau$
(b) $;$ For each $x in X,$ and for each $U in tau$ such that $x in U,$ there exists $V in mathcal B$ such that $x in V$ and $V subseteq U.$




Note: The term "base" is more commonly used than "basis", but I'll use "basis" in agreement with the usage by user 170039.



Roughly speaking, if we consider (open) neighborhoods of $x$ as "measures of being close to $x$", where being closer to $x$ is described by the use of a smaller (in the subset sense) neighborhood of $x,$ then a basis for $tau$ is a subcollection of open sets that is sufficient to describe being arbitrarily close to any given point.



Note that this notion of being close to $x$ has more structure to it than a notion entirely based on the subset relation, because we also have the property that any finite number of "close to $x$ conditions" can be replaced by a single "close to $x$ condition", due to the finite intersection property of open sets. More specifically, being simultaneously $U_1$-close to $x$ and $U_2$-close to $x$ can be replaced by being $(U_1 cap U_2)$-close to $x$ in the case of $tau,$ and by being $V$-close to $x$ for some $mathcal B$-element $V$ such that $V subseteq U overset{text{def}}{=} U_1 cap U_2$ in the case of $mathcal B.$ For example, suppose $X = {x,y,z}$ and $tau = {;{x,y}, ; {y,z};}.$ Note that $tau$ is not a topology on $X.$ Nonetheless, we can still talk about being close to elements of $X.$ For example, there is the notion of being "${x,y}$-close to $y$" and there is the notion of being ${y,z}$-close to $y.$ However, note that there is no single closeness condition that implies each of these conditions, because the only subset of $X$ containing $y$ that is a subset of ${x,y}$ and a subset of ${y,z}$ is ${y},$ and ${y} not in tau.$



A topology can have more than one basis. For example, in the case of the real numbers with its usual topology, the collection of all open intervals of finite length is a basis and the collection of all open intervals of finite length with rational endpoints is a basis.



Question: [This occurred to me when I was writing these notes, and I thought others here might be interested.] How many bases exist for the usual topology of the real numbers? The answer is $2^{c}.$ Since each such basis is a collection of open sets, and there are $2^c$ many collections of open sets of $mathbb R$ (because there are $c$ many open sets of ${mathbb R}),$ it follows that there are at most $2^c$ many bases for the usual topology of the real numbers. The following shows that there are at least $2^c$ many bases for the usual topology of the real numbers. Let $D_1$ and $D_2$ be disjoint subsets of ${mathbb R}$ such that $D_1$ has cardinality $c$ and $D_2$ is dense-in-${mathbb R}$ (for example, $D_1$ could be the set of irrational numbers between $0$ and $1,$ and $D_2$ could be the set of rational numbers), and consider the collection of open intervals each of whose endpoint(s) belongs to $D_1 cup D_{2}.$ This collection of open intervals is a basis for ${mathbb R}.$ Moreover, if we remove from this collection any set consisting only of open intervals each of whose endpoint(s) belongs to $D_{1},$ then what remains will also be a basis, and there are $2^c$ many ways to remove such sets of intervals from the original collection of open intervals (because there are $2^c$ many subsets of the collection of open intervals each of whose endpoint(s) belongs to $D_{1}).$




Theorem 1: Let $(X,tau)$ be a topological space and let $mathcal B$ be a collection of subsets of $X$ such that:



(a) $;;mathcal B subseteq tau$
(b) $;$ Every element of $tau$ is the union of some subcollection of elements in $mathcal B.$



Then $mathcal B$ is a basis for $(X,tau).$




Regarding (b) above, we are using the convention that an empty union of sets is possible, and that an empty union of sets is equal to the empty set.




Theorem 2: Let $(X,tau)$ be a topological space and let $mathcal B$ be a basis for $(X,tau).$ Then every element of $tau$ is the union of some subcollection of elements in $mathcal B.$




Theorems 1 and 2 together imply that we could have defined "$mathcal B$ is a basis for $(X,tau)$" by replacing (b) in our earlier definition with (b) in Theorem 1. This alternate way of defining a basis for $(X,tau)$ is used in some books.



Note that (b) in Theorem 1 provides a way to generate all elements of $tau$ from the elements in $mathcal B$ — take unions. This is analogous to having a way to generate all elements in a vector space from a (vector space) basis — take linear combinations.



2. Given ${mathcal B} subseteq {cal P}(X),$ find a topology ${tau}_{mathcal B}$ on $X$ such that $mathcal B$ is a basis for $(X,tau_{mathcal B}).$



Suppose that no topology on the set $X$ has been provided. Can we obtain a topology ${tau}_{mathcal B}$ on $X$ by picking some collection $mathcal B subseteq {cal P}(X)$ and letting ${tau}_{mathcal B}$ be the collection of all possible unions of elements in $mathcal B$?




Example 1: Let $X = {x,y}$ and let $mathcal B = {; {x};}.$ Then the collection of all possible unions of elements in $mathcal B$ gives ${tau}_{mathcal B} = {; emptyset,;{x};},$ but ${tau}_{mathcal B}$ is not a topology on $X,$ because $X notin {tau}_{mathcal B}.$ (A simpler example is to use $mathcal B = emptyset,$ but I wanted to give a less trivial example.)




Perhaps we can fix the problem that arises in Example 1 by assuming $cup {mathcal B} = X.$ Note that $cup {mathcal B} = X$ is equivalent to the first bullet assumption in user 170039’s statement of the definition of "basis for a topology" from Munkres's book. Example 2 shows that having $cup {mathcal B} = X$ is not sufficient for ${tau}_{mathcal B}$ to be a topology on $X.$




Example 2: Let $X = {x,y,z}$ and let ${mathcal B} = {;{x,y},;{y,z};}.$ Then $cup {mathcal B} = X.$ However, the collection of all possible unions of elements in $mathcal B$ gives ${tau}_{mathcal B} = {;emptyset,; {x,y},;{y,z},; X;},$ and ${tau}_{mathcal B}$ is not a topology because ${tau}_{mathcal B}$ is not closed under finite intersections — note that ${x,y} cap {y,z} = {y}$ and ${y} notin {tau}_{mathcal B}.$




It turns out that fixing both problems, namely the problem that Example 1 illustrates and the problem that Example 2 illustrates, is sufficient for ${tau}_{mathcal B}$ to be a topology on $X.$




Theorem 3: Let $X$ be a set and let $mathcal B$ be a collection of subsets of $X$ such that:



(a) $;; cup {mathcal B} = X$
(b) $;$ For each positive integer $n,$ if $B_1 in mathcal B$ and $B_2 in mathcal B$ and $cdots$ and $B_n in {mathcal B},$ then $B_1 cap B_2 cap cdots cap B_n$ can be written as the union of (possibly infinitely many) elements in ${mathcal B}.$



If we let ${tau}_{mathcal B}$ be the collection of all possible unions of elements in $mathcal B,$ then $(X,{tau}_{mathcal B})$ is a topological space and $mathcal B$ is a basis for $(X,{tau}_{mathcal B}).$




In some books Theorem 3 is buried in results about a subbasis for a topological space. Explicit statements of it can be found in Theorem 3.50 on p. 26 of Introduction to General Topology by Helen Frances Cullen (1968) and in 2.2 Base Characterization on p. 153 of An Introduction to Topology and Homotopy by Allan John Sieradski (1992). On the off-chance that anyone looks at Cullen's book, note that in her book she assumes an empty intersection gives the underlying universal set (see top of p. 416). Since this is not a standard assumption, I've avoided this assumption by including (a).



Theorem 4 is the more commonly stated condition for a collection of subsets of $X$ to generate a topology on $X,$ where the method of generation is by using the collection of all possible unions of elements in that collection of subsets. At the risk of stating the obvious, if $mathcal B$ satisfies the assumptions in Theorem 3 and $mathcal B$ also satisfies the assumptions in Theorem 4 (in fact, one can show that satisfying the assumptions in either theorem implies satisfying the assumptions in the other theorem), then the topology obtained using Theorem 3 is the same as the topology obtained using Theorem 4. This follows from the fact that in each theorem the same procedure is used to obtain the topology, namely the topology is the collection of all possible unions of elements in ${mathcal B}.$




Theorem 4: Let $X$ be a set and let $mathcal B$ be a collection of subsets of $X$ such that:



(a) $;; cup {mathcal B} = X$
(b) $;$ If $U in mathcal B$ and $V in mathcal B$ and $x in U cap V,$ then there exists $W in mathcal B$ such that $x in W$ and $W subseteq U cap V.$



If we let ${tau}_{mathcal B}$ be the collection of all possible unions of elements in $mathcal B,$ then $(X,{tau}_{mathcal B})$ is a topological space and $mathcal B$ is a basis for $(X,{tau}_{mathcal B}).$




One the things that (b) in Theorem 4 does is to incorporate our earlier observation (long paragraph a little below the definition of "basis") that finitely many "close to $x$" conditions can be replaced by a single "close to $x$" condition. Note that in Example 2, ${tau}_{mathcal B}$ has the property that "${x,y}$-close to $y$ and ${y,z}$-close to $y$" cannot be equaled or strengthened by any single ${tau}_{mathcal B}$-closeness notion.



In our earlier vector space analogy, Theorems 3 and 4 are somewhat akin to finding conditions under which a set of vectors is a linearly independent set, then defining the linear span of those vectors, and finally observing that the linear span of those vectors forms a vector space.



3. Comparing $tau$ and $tau_{mathcal B}$ when $(X,tau)$ is a topological space and ${mathcal B} subseteq {cal P}(X)$ is such that $(X,tau_{mathcal B})$ is a topological space.



Let $(X,tau)$ be a topological space and let $mathcal B$ be a collection of subsets of $X$ satisfying the assumptions in either Theorem 3 or Theorem 4 (or both). In this situation we have two topological spaces to contend with, $(X,tau)$ and $(X,,tau_{mathcal B}).$ In general, we have $tau neq tau_{mathcal B}.$ Indeed, there is no reason why we would expect there to be any relation between $tau$ and $tau_{mathcal B},$ aside from the fact that each is a collection of subsets of $X$ that satisfies the axioms for a topology on $X.$




Example 3: Suppose ${mathcal B} not subseteq {tau}.$ Then it is easy to see that $tau neq tau_{mathcal B}.$ (Proof: Let $B in mathcal B$ such that $B not in {tau}.$ Then $B$ belongs to $tau_{mathcal B},$ because we always have ${mathcal B} subseteq tau_{mathcal B},$ and hence $B$ does not belong to ${tau}.$ Therefore, $tau neq tau_{mathcal B}.)$




Example 4, due to Merk Zockerborg, shows that ${mathcal B} subseteq {tau}$ is not sufficient to conclude that $tau = tau_{mathcal B}.$




Example 4: Let $X = {x,y}$ and $tau = {;emptyset,;{x},;{y},;{x,y};}$ and ${mathcal B} = {;{x},;{x,y};}.$ Then ${mathcal B} subseteq {tau}$ and $tau_{mathcal B} = {;emptyset,;{x},;{x,y};}.$ Therefore, $tau neq tau_{mathcal B}.$




Corollary 2 below gives one possible assumption that, along with the assumption ${mathcal B} subseteq {tau},$ is sufficient to conclude that $tau = tau_{mathcal B}.$




Theorem 5: Let $X$ be a set and let ${mathcal B}_1,$ ${mathcal B}_2$ each be collections of subsets of $X$ that satisfy the assumptions in either Theorem 3 or Theorem 4. Then ${tau}_{{mathcal B}_1} subseteq {tau}_{{mathcal B}_2}$ if and only if the following assertion holds:



For each $B_1 in {mathcal B}_1$ and for each $x in B_{1},$ there exists $B_2 in {mathcal B}_2$ such that $x in B_2$ and $B_2 subseteq B_{1}.$




$;$




Corollary 1: Let $X$ be a set and let ${mathcal B}_1,$ ${mathcal B}_2$ each be collections of subsets of $X$ that satisfy the assumptions in either Theorem 3 or Theorem 4. Then ${tau}_{{mathcal B}_1} = {tau}_{{mathcal B}_2}$ if and only if both of the following assertions hold:



For each $B_1 in {mathcal B}_1$ and for each $x in B_{1},$ there exists $B_2 in {mathcal B}_2$ such that $x in B_2$ and $B_2 subseteq B_{1}.$

For each $B_2 in {mathcal B}_2$ and for each $x in B_{2},$ there exists $B_1 in {mathcal B}_1$ such that $x in B_1$ and $B_1 subseteq B_{2}.$




$;$




Corollary 2: Let $(X,tau)$ be a topological space and let $mathcal B$ be a collection of subsets of $X$ that satisfies the assumptions in either Theorem 3 or Theorem 4 such that:



(a) $;; {mathcal B} subseteq {tau}$
(b) $;$ For each $U in tau$ and for each $x in U,$ there exists $B in mathcal B$ such that $x in B$ and $B subseteq U.$



Then $tau = tau_{mathcal B}.$




Proof of Corollary 2: From ${mathcal B} subseteq {tau}$ and the fact that $tau$ is closed under arbitrary unions, it follows that $tau_{mathcal B} subseteq {tau}.$ Applying Theorem 5 with ${mathcal B}_1 = tau$ and ${mathcal B}_2 = {mathcal B},$ and observing that ${tau}_{{mathcal B}_1} = {tau}_{tau} = tau$ (because $tau$ is closed under arbitrary unions), we get $tau subseteq tau_{mathcal B}.$ (The notation is slightly confusing, since in ${tau}_{tau}$ the subscripted $tau$ is a topology on $X$ and the other $tau$ is part of the notation that symbolizes the result of carrying out a certain operation on subscripted set.)






share|cite|improve this answer























  • You are right. That's what we have discussed here and here.
    – user 170039
    Jul 7 '16 at 14:57






  • 1




    sorry to dig up a long inactive post, but I think that the claim $mathcal B subseteq {tau} implies tau = tau_{mathcal B}$ is incorrect? For instance consider the Sierpinski topology on $X={0,1}$, which has $tau_{mathcal B} = { emptyset, {1}, {0, 1} }$ generated by basis sets ${1}$ and ${0,1}$. Consider the discrete topology $tau = { emptyset, {0}, {1}, {0,1} }$. Clearly $mathcal B subset tau$ but $tau_{mathcal B} neq tau$.
    – Merk Zockerborg
    Nov 8 at 14:26








  • 1




    @Merk Zockerborg: I'll look at this tomorrow when I have more time, but for now I think you are correct and more is needed to conclude the two topologies are equal. I think I know what's missing, but I want to preserve the simplicity of my answer (which appears to have been simplified too much!), and to carefully rewrite my answer to incorporate what's needed, it's best that I do it in the morning hours when I'm fresher.
    – Dave L. Renfro
    Nov 8 at 21:17












  • @Merk Zockerborg: It wasn't until yesterday (day after "tomorrow" in my earlier comment) that I began looking at this, and it appears that a very extensive revision is needed. In fact, I'm completely starting all over again. However, given the kinds of confusion this topic tends to generate, and the typically quick way it's taken care of in topology texts, I think it would be useful to have in Stack Exchange a discussion that fleshes out all the various nuances in a careful way. Not sure I'll have it done today, but I'm working some more on it right now.
    – Dave L. Renfro
    Nov 11 at 18:39












  • @Merk Zockerborg: I've just posted a lengthy replacement essay. I mostly finished yesterday (except for two corollaries at the very end), but I wanted to go over everything again, which I did this morning. Ideally, I'd like to give it another "read through and tinkering" tomorrow morning, because there are so many details I could have slipped up on or typos made, but I have some extensive contract work I need to get busy with this week, so I'm posting what I have at this time so I can (hopefully) put it behind me and get to work on the other stuff.
    – Dave L. Renfro
    Nov 12 at 15:07















up vote
6
down vote



accepted







up vote
6
down vote



accepted






This is a completely revised version of my previous answer, which was not correct. The existence of a major error in my previous answer was recently pointed out to me by Merk Zockerborg.



The main result relevant to the question asked by user 170039 is Corollary 2 below.



Contents




  1. Basis for a topological space $(X,tau).$


  2. Given ${mathcal B} subseteq {cal P}(X),$ find a topology ${tau}_{mathcal B}$ on $X$ such that $mathcal B$ is a basis for $(X,tau_{mathcal B}).$


  3. Comparing $tau$ and $tau_{mathcal B}$ when $(X,tau)$ is a topological space and ${mathcal B} subseteq {cal P}(X)$ is such that $(X,tau_{mathcal B})$ is a topological space.



Notation: $X$ is a set and ${cal P}(X)$ is the collection of all subsets of $X.$ The (sometimes subscripted) symbols $tau$ and $mathcal B$ will denote subsets of ${cal P}(X).$ Most of the time (but not always) $tau$ will be a topology on $X$ and $mathcal B$ will be a basis for a topology on $X.$



In the spirit of making this an expository overview of some introductory ideas that arise when introducing the notion of a basis for a topological space, I am not including proofs of the stated theorems. However, those actively studying this topic in topology may find it useful to provide proofs.



1. Basis for a topological space $(X,tau).$




Definition: Let $(X,tau)$ be a topological space. A basis for $(X,tau)$ is a collection $mathcal B$ of subsets of $X$ such that:



(a) $;;mathcal B subseteq tau$
(b) $;$ For each $x in X,$ and for each $U in tau$ such that $x in U,$ there exists $V in mathcal B$ such that $x in V$ and $V subseteq U.$




Note: The term "base" is more commonly used than "basis", but I'll use "basis" in agreement with the usage by user 170039.



Roughly speaking, if we consider (open) neighborhoods of $x$ as "measures of being close to $x$", where being closer to $x$ is described by the use of a smaller (in the subset sense) neighborhood of $x,$ then a basis for $tau$ is a subcollection of open sets that is sufficient to describe being arbitrarily close to any given point.



Note that this notion of being close to $x$ has more structure to it than a notion entirely based on the subset relation, because we also have the property that any finite number of "close to $x$ conditions" can be replaced by a single "close to $x$ condition", due to the finite intersection property of open sets. More specifically, being simultaneously $U_1$-close to $x$ and $U_2$-close to $x$ can be replaced by being $(U_1 cap U_2)$-close to $x$ in the case of $tau,$ and by being $V$-close to $x$ for some $mathcal B$-element $V$ such that $V subseteq U overset{text{def}}{=} U_1 cap U_2$ in the case of $mathcal B.$ For example, suppose $X = {x,y,z}$ and $tau = {;{x,y}, ; {y,z};}.$ Note that $tau$ is not a topology on $X.$ Nonetheless, we can still talk about being close to elements of $X.$ For example, there is the notion of being "${x,y}$-close to $y$" and there is the notion of being ${y,z}$-close to $y.$ However, note that there is no single closeness condition that implies each of these conditions, because the only subset of $X$ containing $y$ that is a subset of ${x,y}$ and a subset of ${y,z}$ is ${y},$ and ${y} not in tau.$



A topology can have more than one basis. For example, in the case of the real numbers with its usual topology, the collection of all open intervals of finite length is a basis and the collection of all open intervals of finite length with rational endpoints is a basis.



Question: [This occurred to me when I was writing these notes, and I thought others here might be interested.] How many bases exist for the usual topology of the real numbers? The answer is $2^{c}.$ Since each such basis is a collection of open sets, and there are $2^c$ many collections of open sets of $mathbb R$ (because there are $c$ many open sets of ${mathbb R}),$ it follows that there are at most $2^c$ many bases for the usual topology of the real numbers. The following shows that there are at least $2^c$ many bases for the usual topology of the real numbers. Let $D_1$ and $D_2$ be disjoint subsets of ${mathbb R}$ such that $D_1$ has cardinality $c$ and $D_2$ is dense-in-${mathbb R}$ (for example, $D_1$ could be the set of irrational numbers between $0$ and $1,$ and $D_2$ could be the set of rational numbers), and consider the collection of open intervals each of whose endpoint(s) belongs to $D_1 cup D_{2}.$ This collection of open intervals is a basis for ${mathbb R}.$ Moreover, if we remove from this collection any set consisting only of open intervals each of whose endpoint(s) belongs to $D_{1},$ then what remains will also be a basis, and there are $2^c$ many ways to remove such sets of intervals from the original collection of open intervals (because there are $2^c$ many subsets of the collection of open intervals each of whose endpoint(s) belongs to $D_{1}).$




Theorem 1: Let $(X,tau)$ be a topological space and let $mathcal B$ be a collection of subsets of $X$ such that:



(a) $;;mathcal B subseteq tau$
(b) $;$ Every element of $tau$ is the union of some subcollection of elements in $mathcal B.$



Then $mathcal B$ is a basis for $(X,tau).$




Regarding (b) above, we are using the convention that an empty union of sets is possible, and that an empty union of sets is equal to the empty set.




Theorem 2: Let $(X,tau)$ be a topological space and let $mathcal B$ be a basis for $(X,tau).$ Then every element of $tau$ is the union of some subcollection of elements in $mathcal B.$




Theorems 1 and 2 together imply that we could have defined "$mathcal B$ is a basis for $(X,tau)$" by replacing (b) in our earlier definition with (b) in Theorem 1. This alternate way of defining a basis for $(X,tau)$ is used in some books.



Note that (b) in Theorem 1 provides a way to generate all elements of $tau$ from the elements in $mathcal B$ — take unions. This is analogous to having a way to generate all elements in a vector space from a (vector space) basis — take linear combinations.



2. Given ${mathcal B} subseteq {cal P}(X),$ find a topology ${tau}_{mathcal B}$ on $X$ such that $mathcal B$ is a basis for $(X,tau_{mathcal B}).$



Suppose that no topology on the set $X$ has been provided. Can we obtain a topology ${tau}_{mathcal B}$ on $X$ by picking some collection $mathcal B subseteq {cal P}(X)$ and letting ${tau}_{mathcal B}$ be the collection of all possible unions of elements in $mathcal B$?




Example 1: Let $X = {x,y}$ and let $mathcal B = {; {x};}.$ Then the collection of all possible unions of elements in $mathcal B$ gives ${tau}_{mathcal B} = {; emptyset,;{x};},$ but ${tau}_{mathcal B}$ is not a topology on $X,$ because $X notin {tau}_{mathcal B}.$ (A simpler example is to use $mathcal B = emptyset,$ but I wanted to give a less trivial example.)




Perhaps we can fix the problem that arises in Example 1 by assuming $cup {mathcal B} = X.$ Note that $cup {mathcal B} = X$ is equivalent to the first bullet assumption in user 170039’s statement of the definition of "basis for a topology" from Munkres's book. Example 2 shows that having $cup {mathcal B} = X$ is not sufficient for ${tau}_{mathcal B}$ to be a topology on $X.$




Example 2: Let $X = {x,y,z}$ and let ${mathcal B} = {;{x,y},;{y,z};}.$ Then $cup {mathcal B} = X.$ However, the collection of all possible unions of elements in $mathcal B$ gives ${tau}_{mathcal B} = {;emptyset,; {x,y},;{y,z},; X;},$ and ${tau}_{mathcal B}$ is not a topology because ${tau}_{mathcal B}$ is not closed under finite intersections — note that ${x,y} cap {y,z} = {y}$ and ${y} notin {tau}_{mathcal B}.$




It turns out that fixing both problems, namely the problem that Example 1 illustrates and the problem that Example 2 illustrates, is sufficient for ${tau}_{mathcal B}$ to be a topology on $X.$




Theorem 3: Let $X$ be a set and let $mathcal B$ be a collection of subsets of $X$ such that:



(a) $;; cup {mathcal B} = X$
(b) $;$ For each positive integer $n,$ if $B_1 in mathcal B$ and $B_2 in mathcal B$ and $cdots$ and $B_n in {mathcal B},$ then $B_1 cap B_2 cap cdots cap B_n$ can be written as the union of (possibly infinitely many) elements in ${mathcal B}.$



If we let ${tau}_{mathcal B}$ be the collection of all possible unions of elements in $mathcal B,$ then $(X,{tau}_{mathcal B})$ is a topological space and $mathcal B$ is a basis for $(X,{tau}_{mathcal B}).$




In some books Theorem 3 is buried in results about a subbasis for a topological space. Explicit statements of it can be found in Theorem 3.50 on p. 26 of Introduction to General Topology by Helen Frances Cullen (1968) and in 2.2 Base Characterization on p. 153 of An Introduction to Topology and Homotopy by Allan John Sieradski (1992). On the off-chance that anyone looks at Cullen's book, note that in her book she assumes an empty intersection gives the underlying universal set (see top of p. 416). Since this is not a standard assumption, I've avoided this assumption by including (a).



Theorem 4 is the more commonly stated condition for a collection of subsets of $X$ to generate a topology on $X,$ where the method of generation is by using the collection of all possible unions of elements in that collection of subsets. At the risk of stating the obvious, if $mathcal B$ satisfies the assumptions in Theorem 3 and $mathcal B$ also satisfies the assumptions in Theorem 4 (in fact, one can show that satisfying the assumptions in either theorem implies satisfying the assumptions in the other theorem), then the topology obtained using Theorem 3 is the same as the topology obtained using Theorem 4. This follows from the fact that in each theorem the same procedure is used to obtain the topology, namely the topology is the collection of all possible unions of elements in ${mathcal B}.$




Theorem 4: Let $X$ be a set and let $mathcal B$ be a collection of subsets of $X$ such that:



(a) $;; cup {mathcal B} = X$
(b) $;$ If $U in mathcal B$ and $V in mathcal B$ and $x in U cap V,$ then there exists $W in mathcal B$ such that $x in W$ and $W subseteq U cap V.$



If we let ${tau}_{mathcal B}$ be the collection of all possible unions of elements in $mathcal B,$ then $(X,{tau}_{mathcal B})$ is a topological space and $mathcal B$ is a basis for $(X,{tau}_{mathcal B}).$




One the things that (b) in Theorem 4 does is to incorporate our earlier observation (long paragraph a little below the definition of "basis") that finitely many "close to $x$" conditions can be replaced by a single "close to $x$" condition. Note that in Example 2, ${tau}_{mathcal B}$ has the property that "${x,y}$-close to $y$ and ${y,z}$-close to $y$" cannot be equaled or strengthened by any single ${tau}_{mathcal B}$-closeness notion.



In our earlier vector space analogy, Theorems 3 and 4 are somewhat akin to finding conditions under which a set of vectors is a linearly independent set, then defining the linear span of those vectors, and finally observing that the linear span of those vectors forms a vector space.



3. Comparing $tau$ and $tau_{mathcal B}$ when $(X,tau)$ is a topological space and ${mathcal B} subseteq {cal P}(X)$ is such that $(X,tau_{mathcal B})$ is a topological space.



Let $(X,tau)$ be a topological space and let $mathcal B$ be a collection of subsets of $X$ satisfying the assumptions in either Theorem 3 or Theorem 4 (or both). In this situation we have two topological spaces to contend with, $(X,tau)$ and $(X,,tau_{mathcal B}).$ In general, we have $tau neq tau_{mathcal B}.$ Indeed, there is no reason why we would expect there to be any relation between $tau$ and $tau_{mathcal B},$ aside from the fact that each is a collection of subsets of $X$ that satisfies the axioms for a topology on $X.$




Example 3: Suppose ${mathcal B} not subseteq {tau}.$ Then it is easy to see that $tau neq tau_{mathcal B}.$ (Proof: Let $B in mathcal B$ such that $B not in {tau}.$ Then $B$ belongs to $tau_{mathcal B},$ because we always have ${mathcal B} subseteq tau_{mathcal B},$ and hence $B$ does not belong to ${tau}.$ Therefore, $tau neq tau_{mathcal B}.)$




Example 4, due to Merk Zockerborg, shows that ${mathcal B} subseteq {tau}$ is not sufficient to conclude that $tau = tau_{mathcal B}.$




Example 4: Let $X = {x,y}$ and $tau = {;emptyset,;{x},;{y},;{x,y};}$ and ${mathcal B} = {;{x},;{x,y};}.$ Then ${mathcal B} subseteq {tau}$ and $tau_{mathcal B} = {;emptyset,;{x},;{x,y};}.$ Therefore, $tau neq tau_{mathcal B}.$




Corollary 2 below gives one possible assumption that, along with the assumption ${mathcal B} subseteq {tau},$ is sufficient to conclude that $tau = tau_{mathcal B}.$




Theorem 5: Let $X$ be a set and let ${mathcal B}_1,$ ${mathcal B}_2$ each be collections of subsets of $X$ that satisfy the assumptions in either Theorem 3 or Theorem 4. Then ${tau}_{{mathcal B}_1} subseteq {tau}_{{mathcal B}_2}$ if and only if the following assertion holds:



For each $B_1 in {mathcal B}_1$ and for each $x in B_{1},$ there exists $B_2 in {mathcal B}_2$ such that $x in B_2$ and $B_2 subseteq B_{1}.$




$;$




Corollary 1: Let $X$ be a set and let ${mathcal B}_1,$ ${mathcal B}_2$ each be collections of subsets of $X$ that satisfy the assumptions in either Theorem 3 or Theorem 4. Then ${tau}_{{mathcal B}_1} = {tau}_{{mathcal B}_2}$ if and only if both of the following assertions hold:



For each $B_1 in {mathcal B}_1$ and for each $x in B_{1},$ there exists $B_2 in {mathcal B}_2$ such that $x in B_2$ and $B_2 subseteq B_{1}.$

For each $B_2 in {mathcal B}_2$ and for each $x in B_{2},$ there exists $B_1 in {mathcal B}_1$ such that $x in B_1$ and $B_1 subseteq B_{2}.$




$;$




Corollary 2: Let $(X,tau)$ be a topological space and let $mathcal B$ be a collection of subsets of $X$ that satisfies the assumptions in either Theorem 3 or Theorem 4 such that:



(a) $;; {mathcal B} subseteq {tau}$
(b) $;$ For each $U in tau$ and for each $x in U,$ there exists $B in mathcal B$ such that $x in B$ and $B subseteq U.$



Then $tau = tau_{mathcal B}.$




Proof of Corollary 2: From ${mathcal B} subseteq {tau}$ and the fact that $tau$ is closed under arbitrary unions, it follows that $tau_{mathcal B} subseteq {tau}.$ Applying Theorem 5 with ${mathcal B}_1 = tau$ and ${mathcal B}_2 = {mathcal B},$ and observing that ${tau}_{{mathcal B}_1} = {tau}_{tau} = tau$ (because $tau$ is closed under arbitrary unions), we get $tau subseteq tau_{mathcal B}.$ (The notation is slightly confusing, since in ${tau}_{tau}$ the subscripted $tau$ is a topology on $X$ and the other $tau$ is part of the notation that symbolizes the result of carrying out a certain operation on subscripted set.)






share|cite|improve this answer














This is a completely revised version of my previous answer, which was not correct. The existence of a major error in my previous answer was recently pointed out to me by Merk Zockerborg.



The main result relevant to the question asked by user 170039 is Corollary 2 below.



Contents




  1. Basis for a topological space $(X,tau).$


  2. Given ${mathcal B} subseteq {cal P}(X),$ find a topology ${tau}_{mathcal B}$ on $X$ such that $mathcal B$ is a basis for $(X,tau_{mathcal B}).$


  3. Comparing $tau$ and $tau_{mathcal B}$ when $(X,tau)$ is a topological space and ${mathcal B} subseteq {cal P}(X)$ is such that $(X,tau_{mathcal B})$ is a topological space.



Notation: $X$ is a set and ${cal P}(X)$ is the collection of all subsets of $X.$ The (sometimes subscripted) symbols $tau$ and $mathcal B$ will denote subsets of ${cal P}(X).$ Most of the time (but not always) $tau$ will be a topology on $X$ and $mathcal B$ will be a basis for a topology on $X.$



In the spirit of making this an expository overview of some introductory ideas that arise when introducing the notion of a basis for a topological space, I am not including proofs of the stated theorems. However, those actively studying this topic in topology may find it useful to provide proofs.



1. Basis for a topological space $(X,tau).$




Definition: Let $(X,tau)$ be a topological space. A basis for $(X,tau)$ is a collection $mathcal B$ of subsets of $X$ such that:



(a) $;;mathcal B subseteq tau$
(b) $;$ For each $x in X,$ and for each $U in tau$ such that $x in U,$ there exists $V in mathcal B$ such that $x in V$ and $V subseteq U.$




Note: The term "base" is more commonly used than "basis", but I'll use "basis" in agreement with the usage by user 170039.



Roughly speaking, if we consider (open) neighborhoods of $x$ as "measures of being close to $x$", where being closer to $x$ is described by the use of a smaller (in the subset sense) neighborhood of $x,$ then a basis for $tau$ is a subcollection of open sets that is sufficient to describe being arbitrarily close to any given point.



Note that this notion of being close to $x$ has more structure to it than a notion entirely based on the subset relation, because we also have the property that any finite number of "close to $x$ conditions" can be replaced by a single "close to $x$ condition", due to the finite intersection property of open sets. More specifically, being simultaneously $U_1$-close to $x$ and $U_2$-close to $x$ can be replaced by being $(U_1 cap U_2)$-close to $x$ in the case of $tau,$ and by being $V$-close to $x$ for some $mathcal B$-element $V$ such that $V subseteq U overset{text{def}}{=} U_1 cap U_2$ in the case of $mathcal B.$ For example, suppose $X = {x,y,z}$ and $tau = {;{x,y}, ; {y,z};}.$ Note that $tau$ is not a topology on $X.$ Nonetheless, we can still talk about being close to elements of $X.$ For example, there is the notion of being "${x,y}$-close to $y$" and there is the notion of being ${y,z}$-close to $y.$ However, note that there is no single closeness condition that implies each of these conditions, because the only subset of $X$ containing $y$ that is a subset of ${x,y}$ and a subset of ${y,z}$ is ${y},$ and ${y} not in tau.$



A topology can have more than one basis. For example, in the case of the real numbers with its usual topology, the collection of all open intervals of finite length is a basis and the collection of all open intervals of finite length with rational endpoints is a basis.



Question: [This occurred to me when I was writing these notes, and I thought others here might be interested.] How many bases exist for the usual topology of the real numbers? The answer is $2^{c}.$ Since each such basis is a collection of open sets, and there are $2^c$ many collections of open sets of $mathbb R$ (because there are $c$ many open sets of ${mathbb R}),$ it follows that there are at most $2^c$ many bases for the usual topology of the real numbers. The following shows that there are at least $2^c$ many bases for the usual topology of the real numbers. Let $D_1$ and $D_2$ be disjoint subsets of ${mathbb R}$ such that $D_1$ has cardinality $c$ and $D_2$ is dense-in-${mathbb R}$ (for example, $D_1$ could be the set of irrational numbers between $0$ and $1,$ and $D_2$ could be the set of rational numbers), and consider the collection of open intervals each of whose endpoint(s) belongs to $D_1 cup D_{2}.$ This collection of open intervals is a basis for ${mathbb R}.$ Moreover, if we remove from this collection any set consisting only of open intervals each of whose endpoint(s) belongs to $D_{1},$ then what remains will also be a basis, and there are $2^c$ many ways to remove such sets of intervals from the original collection of open intervals (because there are $2^c$ many subsets of the collection of open intervals each of whose endpoint(s) belongs to $D_{1}).$




Theorem 1: Let $(X,tau)$ be a topological space and let $mathcal B$ be a collection of subsets of $X$ such that:



(a) $;;mathcal B subseteq tau$
(b) $;$ Every element of $tau$ is the union of some subcollection of elements in $mathcal B.$



Then $mathcal B$ is a basis for $(X,tau).$




Regarding (b) above, we are using the convention that an empty union of sets is possible, and that an empty union of sets is equal to the empty set.




Theorem 2: Let $(X,tau)$ be a topological space and let $mathcal B$ be a basis for $(X,tau).$ Then every element of $tau$ is the union of some subcollection of elements in $mathcal B.$




Theorems 1 and 2 together imply that we could have defined "$mathcal B$ is a basis for $(X,tau)$" by replacing (b) in our earlier definition with (b) in Theorem 1. This alternate way of defining a basis for $(X,tau)$ is used in some books.



Note that (b) in Theorem 1 provides a way to generate all elements of $tau$ from the elements in $mathcal B$ — take unions. This is analogous to having a way to generate all elements in a vector space from a (vector space) basis — take linear combinations.



2. Given ${mathcal B} subseteq {cal P}(X),$ find a topology ${tau}_{mathcal B}$ on $X$ such that $mathcal B$ is a basis for $(X,tau_{mathcal B}).$



Suppose that no topology on the set $X$ has been provided. Can we obtain a topology ${tau}_{mathcal B}$ on $X$ by picking some collection $mathcal B subseteq {cal P}(X)$ and letting ${tau}_{mathcal B}$ be the collection of all possible unions of elements in $mathcal B$?




Example 1: Let $X = {x,y}$ and let $mathcal B = {; {x};}.$ Then the collection of all possible unions of elements in $mathcal B$ gives ${tau}_{mathcal B} = {; emptyset,;{x};},$ but ${tau}_{mathcal B}$ is not a topology on $X,$ because $X notin {tau}_{mathcal B}.$ (A simpler example is to use $mathcal B = emptyset,$ but I wanted to give a less trivial example.)




Perhaps we can fix the problem that arises in Example 1 by assuming $cup {mathcal B} = X.$ Note that $cup {mathcal B} = X$ is equivalent to the first bullet assumption in user 170039’s statement of the definition of "basis for a topology" from Munkres's book. Example 2 shows that having $cup {mathcal B} = X$ is not sufficient for ${tau}_{mathcal B}$ to be a topology on $X.$




Example 2: Let $X = {x,y,z}$ and let ${mathcal B} = {;{x,y},;{y,z};}.$ Then $cup {mathcal B} = X.$ However, the collection of all possible unions of elements in $mathcal B$ gives ${tau}_{mathcal B} = {;emptyset,; {x,y},;{y,z},; X;},$ and ${tau}_{mathcal B}$ is not a topology because ${tau}_{mathcal B}$ is not closed under finite intersections — note that ${x,y} cap {y,z} = {y}$ and ${y} notin {tau}_{mathcal B}.$




It turns out that fixing both problems, namely the problem that Example 1 illustrates and the problem that Example 2 illustrates, is sufficient for ${tau}_{mathcal B}$ to be a topology on $X.$




Theorem 3: Let $X$ be a set and let $mathcal B$ be a collection of subsets of $X$ such that:



(a) $;; cup {mathcal B} = X$
(b) $;$ For each positive integer $n,$ if $B_1 in mathcal B$ and $B_2 in mathcal B$ and $cdots$ and $B_n in {mathcal B},$ then $B_1 cap B_2 cap cdots cap B_n$ can be written as the union of (possibly infinitely many) elements in ${mathcal B}.$



If we let ${tau}_{mathcal B}$ be the collection of all possible unions of elements in $mathcal B,$ then $(X,{tau}_{mathcal B})$ is a topological space and $mathcal B$ is a basis for $(X,{tau}_{mathcal B}).$




In some books Theorem 3 is buried in results about a subbasis for a topological space. Explicit statements of it can be found in Theorem 3.50 on p. 26 of Introduction to General Topology by Helen Frances Cullen (1968) and in 2.2 Base Characterization on p. 153 of An Introduction to Topology and Homotopy by Allan John Sieradski (1992). On the off-chance that anyone looks at Cullen's book, note that in her book she assumes an empty intersection gives the underlying universal set (see top of p. 416). Since this is not a standard assumption, I've avoided this assumption by including (a).



Theorem 4 is the more commonly stated condition for a collection of subsets of $X$ to generate a topology on $X,$ where the method of generation is by using the collection of all possible unions of elements in that collection of subsets. At the risk of stating the obvious, if $mathcal B$ satisfies the assumptions in Theorem 3 and $mathcal B$ also satisfies the assumptions in Theorem 4 (in fact, one can show that satisfying the assumptions in either theorem implies satisfying the assumptions in the other theorem), then the topology obtained using Theorem 3 is the same as the topology obtained using Theorem 4. This follows from the fact that in each theorem the same procedure is used to obtain the topology, namely the topology is the collection of all possible unions of elements in ${mathcal B}.$




Theorem 4: Let $X$ be a set and let $mathcal B$ be a collection of subsets of $X$ such that:



(a) $;; cup {mathcal B} = X$
(b) $;$ If $U in mathcal B$ and $V in mathcal B$ and $x in U cap V,$ then there exists $W in mathcal B$ such that $x in W$ and $W subseteq U cap V.$



If we let ${tau}_{mathcal B}$ be the collection of all possible unions of elements in $mathcal B,$ then $(X,{tau}_{mathcal B})$ is a topological space and $mathcal B$ is a basis for $(X,{tau}_{mathcal B}).$




One the things that (b) in Theorem 4 does is to incorporate our earlier observation (long paragraph a little below the definition of "basis") that finitely many "close to $x$" conditions can be replaced by a single "close to $x$" condition. Note that in Example 2, ${tau}_{mathcal B}$ has the property that "${x,y}$-close to $y$ and ${y,z}$-close to $y$" cannot be equaled or strengthened by any single ${tau}_{mathcal B}$-closeness notion.



In our earlier vector space analogy, Theorems 3 and 4 are somewhat akin to finding conditions under which a set of vectors is a linearly independent set, then defining the linear span of those vectors, and finally observing that the linear span of those vectors forms a vector space.



3. Comparing $tau$ and $tau_{mathcal B}$ when $(X,tau)$ is a topological space and ${mathcal B} subseteq {cal P}(X)$ is such that $(X,tau_{mathcal B})$ is a topological space.



Let $(X,tau)$ be a topological space and let $mathcal B$ be a collection of subsets of $X$ satisfying the assumptions in either Theorem 3 or Theorem 4 (or both). In this situation we have two topological spaces to contend with, $(X,tau)$ and $(X,,tau_{mathcal B}).$ In general, we have $tau neq tau_{mathcal B}.$ Indeed, there is no reason why we would expect there to be any relation between $tau$ and $tau_{mathcal B},$ aside from the fact that each is a collection of subsets of $X$ that satisfies the axioms for a topology on $X.$




Example 3: Suppose ${mathcal B} not subseteq {tau}.$ Then it is easy to see that $tau neq tau_{mathcal B}.$ (Proof: Let $B in mathcal B$ such that $B not in {tau}.$ Then $B$ belongs to $tau_{mathcal B},$ because we always have ${mathcal B} subseteq tau_{mathcal B},$ and hence $B$ does not belong to ${tau}.$ Therefore, $tau neq tau_{mathcal B}.)$




Example 4, due to Merk Zockerborg, shows that ${mathcal B} subseteq {tau}$ is not sufficient to conclude that $tau = tau_{mathcal B}.$




Example 4: Let $X = {x,y}$ and $tau = {;emptyset,;{x},;{y},;{x,y};}$ and ${mathcal B} = {;{x},;{x,y};}.$ Then ${mathcal B} subseteq {tau}$ and $tau_{mathcal B} = {;emptyset,;{x},;{x,y};}.$ Therefore, $tau neq tau_{mathcal B}.$




Corollary 2 below gives one possible assumption that, along with the assumption ${mathcal B} subseteq {tau},$ is sufficient to conclude that $tau = tau_{mathcal B}.$




Theorem 5: Let $X$ be a set and let ${mathcal B}_1,$ ${mathcal B}_2$ each be collections of subsets of $X$ that satisfy the assumptions in either Theorem 3 or Theorem 4. Then ${tau}_{{mathcal B}_1} subseteq {tau}_{{mathcal B}_2}$ if and only if the following assertion holds:



For each $B_1 in {mathcal B}_1$ and for each $x in B_{1},$ there exists $B_2 in {mathcal B}_2$ such that $x in B_2$ and $B_2 subseteq B_{1}.$




$;$




Corollary 1: Let $X$ be a set and let ${mathcal B}_1,$ ${mathcal B}_2$ each be collections of subsets of $X$ that satisfy the assumptions in either Theorem 3 or Theorem 4. Then ${tau}_{{mathcal B}_1} = {tau}_{{mathcal B}_2}$ if and only if both of the following assertions hold:



For each $B_1 in {mathcal B}_1$ and for each $x in B_{1},$ there exists $B_2 in {mathcal B}_2$ such that $x in B_2$ and $B_2 subseteq B_{1}.$

For each $B_2 in {mathcal B}_2$ and for each $x in B_{2},$ there exists $B_1 in {mathcal B}_1$ such that $x in B_1$ and $B_1 subseteq B_{2}.$




$;$




Corollary 2: Let $(X,tau)$ be a topological space and let $mathcal B$ be a collection of subsets of $X$ that satisfies the assumptions in either Theorem 3 or Theorem 4 such that:



(a) $;; {mathcal B} subseteq {tau}$
(b) $;$ For each $U in tau$ and for each $x in U,$ there exists $B in mathcal B$ such that $x in B$ and $B subseteq U.$



Then $tau = tau_{mathcal B}.$




Proof of Corollary 2: From ${mathcal B} subseteq {tau}$ and the fact that $tau$ is closed under arbitrary unions, it follows that $tau_{mathcal B} subseteq {tau}.$ Applying Theorem 5 with ${mathcal B}_1 = tau$ and ${mathcal B}_2 = {mathcal B},$ and observing that ${tau}_{{mathcal B}_1} = {tau}_{tau} = tau$ (because $tau$ is closed under arbitrary unions), we get $tau subseteq tau_{mathcal B}.$ (The notation is slightly confusing, since in ${tau}_{tau}$ the subscripted $tau$ is a topology on $X$ and the other $tau$ is part of the notation that symbolizes the result of carrying out a certain operation on subscripted set.)







share|cite|improve this answer














share|cite|improve this answer



share|cite|improve this answer








edited Nov 16 at 6:24

























answered Jul 7 '16 at 14:52









Dave L. Renfro

23.4k33979




23.4k33979












  • You are right. That's what we have discussed here and here.
    – user 170039
    Jul 7 '16 at 14:57






  • 1




    sorry to dig up a long inactive post, but I think that the claim $mathcal B subseteq {tau} implies tau = tau_{mathcal B}$ is incorrect? For instance consider the Sierpinski topology on $X={0,1}$, which has $tau_{mathcal B} = { emptyset, {1}, {0, 1} }$ generated by basis sets ${1}$ and ${0,1}$. Consider the discrete topology $tau = { emptyset, {0}, {1}, {0,1} }$. Clearly $mathcal B subset tau$ but $tau_{mathcal B} neq tau$.
    – Merk Zockerborg
    Nov 8 at 14:26








  • 1




    @Merk Zockerborg: I'll look at this tomorrow when I have more time, but for now I think you are correct and more is needed to conclude the two topologies are equal. I think I know what's missing, but I want to preserve the simplicity of my answer (which appears to have been simplified too much!), and to carefully rewrite my answer to incorporate what's needed, it's best that I do it in the morning hours when I'm fresher.
    – Dave L. Renfro
    Nov 8 at 21:17












  • @Merk Zockerborg: It wasn't until yesterday (day after "tomorrow" in my earlier comment) that I began looking at this, and it appears that a very extensive revision is needed. In fact, I'm completely starting all over again. However, given the kinds of confusion this topic tends to generate, and the typically quick way it's taken care of in topology texts, I think it would be useful to have in Stack Exchange a discussion that fleshes out all the various nuances in a careful way. Not sure I'll have it done today, but I'm working some more on it right now.
    – Dave L. Renfro
    Nov 11 at 18:39












  • @Merk Zockerborg: I've just posted a lengthy replacement essay. I mostly finished yesterday (except for two corollaries at the very end), but I wanted to go over everything again, which I did this morning. Ideally, I'd like to give it another "read through and tinkering" tomorrow morning, because there are so many details I could have slipped up on or typos made, but I have some extensive contract work I need to get busy with this week, so I'm posting what I have at this time so I can (hopefully) put it behind me and get to work on the other stuff.
    – Dave L. Renfro
    Nov 12 at 15:07




















  • You are right. That's what we have discussed here and here.
    – user 170039
    Jul 7 '16 at 14:57






  • 1




    sorry to dig up a long inactive post, but I think that the claim $mathcal B subseteq {tau} implies tau = tau_{mathcal B}$ is incorrect? For instance consider the Sierpinski topology on $X={0,1}$, which has $tau_{mathcal B} = { emptyset, {1}, {0, 1} }$ generated by basis sets ${1}$ and ${0,1}$. Consider the discrete topology $tau = { emptyset, {0}, {1}, {0,1} }$. Clearly $mathcal B subset tau$ but $tau_{mathcal B} neq tau$.
    – Merk Zockerborg
    Nov 8 at 14:26








  • 1




    @Merk Zockerborg: I'll look at this tomorrow when I have more time, but for now I think you are correct and more is needed to conclude the two topologies are equal. I think I know what's missing, but I want to preserve the simplicity of my answer (which appears to have been simplified too much!), and to carefully rewrite my answer to incorporate what's needed, it's best that I do it in the morning hours when I'm fresher.
    – Dave L. Renfro
    Nov 8 at 21:17












  • @Merk Zockerborg: It wasn't until yesterday (day after "tomorrow" in my earlier comment) that I began looking at this, and it appears that a very extensive revision is needed. In fact, I'm completely starting all over again. However, given the kinds of confusion this topic tends to generate, and the typically quick way it's taken care of in topology texts, I think it would be useful to have in Stack Exchange a discussion that fleshes out all the various nuances in a careful way. Not sure I'll have it done today, but I'm working some more on it right now.
    – Dave L. Renfro
    Nov 11 at 18:39












  • @Merk Zockerborg: I've just posted a lengthy replacement essay. I mostly finished yesterday (except for two corollaries at the very end), but I wanted to go over everything again, which I did this morning. Ideally, I'd like to give it another "read through and tinkering" tomorrow morning, because there are so many details I could have slipped up on or typos made, but I have some extensive contract work I need to get busy with this week, so I'm posting what I have at this time so I can (hopefully) put it behind me and get to work on the other stuff.
    – Dave L. Renfro
    Nov 12 at 15:07


















You are right. That's what we have discussed here and here.
– user 170039
Jul 7 '16 at 14:57




You are right. That's what we have discussed here and here.
– user 170039
Jul 7 '16 at 14:57




1




1




sorry to dig up a long inactive post, but I think that the claim $mathcal B subseteq {tau} implies tau = tau_{mathcal B}$ is incorrect? For instance consider the Sierpinski topology on $X={0,1}$, which has $tau_{mathcal B} = { emptyset, {1}, {0, 1} }$ generated by basis sets ${1}$ and ${0,1}$. Consider the discrete topology $tau = { emptyset, {0}, {1}, {0,1} }$. Clearly $mathcal B subset tau$ but $tau_{mathcal B} neq tau$.
– Merk Zockerborg
Nov 8 at 14:26






sorry to dig up a long inactive post, but I think that the claim $mathcal B subseteq {tau} implies tau = tau_{mathcal B}$ is incorrect? For instance consider the Sierpinski topology on $X={0,1}$, which has $tau_{mathcal B} = { emptyset, {1}, {0, 1} }$ generated by basis sets ${1}$ and ${0,1}$. Consider the discrete topology $tau = { emptyset, {0}, {1}, {0,1} }$. Clearly $mathcal B subset tau$ but $tau_{mathcal B} neq tau$.
– Merk Zockerborg
Nov 8 at 14:26






1




1




@Merk Zockerborg: I'll look at this tomorrow when I have more time, but for now I think you are correct and more is needed to conclude the two topologies are equal. I think I know what's missing, but I want to preserve the simplicity of my answer (which appears to have been simplified too much!), and to carefully rewrite my answer to incorporate what's needed, it's best that I do it in the morning hours when I'm fresher.
– Dave L. Renfro
Nov 8 at 21:17






@Merk Zockerborg: I'll look at this tomorrow when I have more time, but for now I think you are correct and more is needed to conclude the two topologies are equal. I think I know what's missing, but I want to preserve the simplicity of my answer (which appears to have been simplified too much!), and to carefully rewrite my answer to incorporate what's needed, it's best that I do it in the morning hours when I'm fresher.
– Dave L. Renfro
Nov 8 at 21:17














@Merk Zockerborg: It wasn't until yesterday (day after "tomorrow" in my earlier comment) that I began looking at this, and it appears that a very extensive revision is needed. In fact, I'm completely starting all over again. However, given the kinds of confusion this topic tends to generate, and the typically quick way it's taken care of in topology texts, I think it would be useful to have in Stack Exchange a discussion that fleshes out all the various nuances in a careful way. Not sure I'll have it done today, but I'm working some more on it right now.
– Dave L. Renfro
Nov 11 at 18:39






@Merk Zockerborg: It wasn't until yesterday (day after "tomorrow" in my earlier comment) that I began looking at this, and it appears that a very extensive revision is needed. In fact, I'm completely starting all over again. However, given the kinds of confusion this topic tends to generate, and the typically quick way it's taken care of in topology texts, I think it would be useful to have in Stack Exchange a discussion that fleshes out all the various nuances in a careful way. Not sure I'll have it done today, but I'm working some more on it right now.
– Dave L. Renfro
Nov 11 at 18:39














@Merk Zockerborg: I've just posted a lengthy replacement essay. I mostly finished yesterday (except for two corollaries at the very end), but I wanted to go over everything again, which I did this morning. Ideally, I'd like to give it another "read through and tinkering" tomorrow morning, because there are so many details I could have slipped up on or typos made, but I have some extensive contract work I need to get busy with this week, so I'm posting what I have at this time so I can (hopefully) put it behind me and get to work on the other stuff.
– Dave L. Renfro
Nov 12 at 15:07






@Merk Zockerborg: I've just posted a lengthy replacement essay. I mostly finished yesterday (except for two corollaries at the very end), but I wanted to go over everything again, which I did this morning. Ideally, I'd like to give it another "read through and tinkering" tomorrow morning, because there are so many details I could have slipped up on or typos made, but I have some extensive contract work I need to get busy with this week, so I'm posting what I have at this time so I can (hopefully) put it behind me and get to work on the other stuff.
– Dave L. Renfro
Nov 12 at 15:07












up vote
2
down vote













You don't need a topology to define a base for a topology, but every basis generates a topology, and you can go about finding bases for existing topologies.



The topology generated by such a basis is determined in the following way:



A subset $mathcal O$ of $X$ is open if for all $xin mathcal O$, there is an element $mathcal B$ of the basis such that $xin mathcal Bsubseteq mathcal O$.



Expressed another way, you can say that the open subsets are the ones which are unions of basis elements.



Why use bases? Well, just like in liner algebra, it's a way to compress information about all open sets into a smaller collection that recovers everything else. For example, the open balls of rational radius around points in $mathbb R^2$ form a countable basis of the ordinary topology on $Bbb R^2$. This collection is not closed under intersection or union, and the full topology on $mathbb R$ contains uncountably many open sets.






share|cite|improve this answer























  • How does this answer my question?
    – user 170039
    Jul 7 '16 at 13:56










  • I guess I should have said (which I have now inserted) that it is not necessary to have a topology before defining a basis. You can define a basis then generate a topology, or else you can find a topology for an existing basis.
    – rschwieb
    Jul 7 '16 at 13:59










  • So by "basis for a topology.." he means "basis for some topology (yet to be determined)", am I right?
    – user 170039
    Jul 7 '16 at 14:04










  • @user170039 It can be: if you define a basis according to these axioms, you can generate a topology. But sometimes it goes the other way where you have a topology and you judiciously choose a basis for it to simplify your proofs.
    – rschwieb
    Jul 7 '16 at 14:08












  • The problem is that suppose we have a topology on $X$, say $mathcal{T}$. Then from the definition, it doesn't follow that a basis for $mathcal{T}$, say $mathcal{B}$ is a subset of $mathcal{T}$. To be precise, let $X=mathbb{R}$ with the standard topology. Then observe that both the set $mathcal{B}_{1}:={(a,b):a,binmathbb{R}}$ and $mathcal{B}_{1}:={[a,b]:a,binmathbb{R}}$ can be considered as a basis for the usual topology on $mathbb{R}$ according to the definition.
    – user 170039
    Jul 7 '16 at 14:14















up vote
2
down vote













You don't need a topology to define a base for a topology, but every basis generates a topology, and you can go about finding bases for existing topologies.



The topology generated by such a basis is determined in the following way:



A subset $mathcal O$ of $X$ is open if for all $xin mathcal O$, there is an element $mathcal B$ of the basis such that $xin mathcal Bsubseteq mathcal O$.



Expressed another way, you can say that the open subsets are the ones which are unions of basis elements.



Why use bases? Well, just like in liner algebra, it's a way to compress information about all open sets into a smaller collection that recovers everything else. For example, the open balls of rational radius around points in $mathbb R^2$ form a countable basis of the ordinary topology on $Bbb R^2$. This collection is not closed under intersection or union, and the full topology on $mathbb R$ contains uncountably many open sets.






share|cite|improve this answer























  • How does this answer my question?
    – user 170039
    Jul 7 '16 at 13:56










  • I guess I should have said (which I have now inserted) that it is not necessary to have a topology before defining a basis. You can define a basis then generate a topology, or else you can find a topology for an existing basis.
    – rschwieb
    Jul 7 '16 at 13:59










  • So by "basis for a topology.." he means "basis for some topology (yet to be determined)", am I right?
    – user 170039
    Jul 7 '16 at 14:04










  • @user170039 It can be: if you define a basis according to these axioms, you can generate a topology. But sometimes it goes the other way where you have a topology and you judiciously choose a basis for it to simplify your proofs.
    – rschwieb
    Jul 7 '16 at 14:08












  • The problem is that suppose we have a topology on $X$, say $mathcal{T}$. Then from the definition, it doesn't follow that a basis for $mathcal{T}$, say $mathcal{B}$ is a subset of $mathcal{T}$. To be precise, let $X=mathbb{R}$ with the standard topology. Then observe that both the set $mathcal{B}_{1}:={(a,b):a,binmathbb{R}}$ and $mathcal{B}_{1}:={[a,b]:a,binmathbb{R}}$ can be considered as a basis for the usual topology on $mathbb{R}$ according to the definition.
    – user 170039
    Jul 7 '16 at 14:14













up vote
2
down vote










up vote
2
down vote









You don't need a topology to define a base for a topology, but every basis generates a topology, and you can go about finding bases for existing topologies.



The topology generated by such a basis is determined in the following way:



A subset $mathcal O$ of $X$ is open if for all $xin mathcal O$, there is an element $mathcal B$ of the basis such that $xin mathcal Bsubseteq mathcal O$.



Expressed another way, you can say that the open subsets are the ones which are unions of basis elements.



Why use bases? Well, just like in liner algebra, it's a way to compress information about all open sets into a smaller collection that recovers everything else. For example, the open balls of rational radius around points in $mathbb R^2$ form a countable basis of the ordinary topology on $Bbb R^2$. This collection is not closed under intersection or union, and the full topology on $mathbb R$ contains uncountably many open sets.






share|cite|improve this answer














You don't need a topology to define a base for a topology, but every basis generates a topology, and you can go about finding bases for existing topologies.



The topology generated by such a basis is determined in the following way:



A subset $mathcal O$ of $X$ is open if for all $xin mathcal O$, there is an element $mathcal B$ of the basis such that $xin mathcal Bsubseteq mathcal O$.



Expressed another way, you can say that the open subsets are the ones which are unions of basis elements.



Why use bases? Well, just like in liner algebra, it's a way to compress information about all open sets into a smaller collection that recovers everything else. For example, the open balls of rational radius around points in $mathbb R^2$ form a countable basis of the ordinary topology on $Bbb R^2$. This collection is not closed under intersection or union, and the full topology on $mathbb R$ contains uncountably many open sets.







share|cite|improve this answer














share|cite|improve this answer



share|cite|improve this answer








edited Jul 7 '16 at 13:57

























answered Jul 7 '16 at 13:56









rschwieb

103k1299238




103k1299238












  • How does this answer my question?
    – user 170039
    Jul 7 '16 at 13:56










  • I guess I should have said (which I have now inserted) that it is not necessary to have a topology before defining a basis. You can define a basis then generate a topology, or else you can find a topology for an existing basis.
    – rschwieb
    Jul 7 '16 at 13:59










  • So by "basis for a topology.." he means "basis for some topology (yet to be determined)", am I right?
    – user 170039
    Jul 7 '16 at 14:04










  • @user170039 It can be: if you define a basis according to these axioms, you can generate a topology. But sometimes it goes the other way where you have a topology and you judiciously choose a basis for it to simplify your proofs.
    – rschwieb
    Jul 7 '16 at 14:08












  • The problem is that suppose we have a topology on $X$, say $mathcal{T}$. Then from the definition, it doesn't follow that a basis for $mathcal{T}$, say $mathcal{B}$ is a subset of $mathcal{T}$. To be precise, let $X=mathbb{R}$ with the standard topology. Then observe that both the set $mathcal{B}_{1}:={(a,b):a,binmathbb{R}}$ and $mathcal{B}_{1}:={[a,b]:a,binmathbb{R}}$ can be considered as a basis for the usual topology on $mathbb{R}$ according to the definition.
    – user 170039
    Jul 7 '16 at 14:14


















  • How does this answer my question?
    – user 170039
    Jul 7 '16 at 13:56










  • I guess I should have said (which I have now inserted) that it is not necessary to have a topology before defining a basis. You can define a basis then generate a topology, or else you can find a topology for an existing basis.
    – rschwieb
    Jul 7 '16 at 13:59










  • So by "basis for a topology.." he means "basis for some topology (yet to be determined)", am I right?
    – user 170039
    Jul 7 '16 at 14:04










  • @user170039 It can be: if you define a basis according to these axioms, you can generate a topology. But sometimes it goes the other way where you have a topology and you judiciously choose a basis for it to simplify your proofs.
    – rschwieb
    Jul 7 '16 at 14:08












  • The problem is that suppose we have a topology on $X$, say $mathcal{T}$. Then from the definition, it doesn't follow that a basis for $mathcal{T}$, say $mathcal{B}$ is a subset of $mathcal{T}$. To be precise, let $X=mathbb{R}$ with the standard topology. Then observe that both the set $mathcal{B}_{1}:={(a,b):a,binmathbb{R}}$ and $mathcal{B}_{1}:={[a,b]:a,binmathbb{R}}$ can be considered as a basis for the usual topology on $mathbb{R}$ according to the definition.
    – user 170039
    Jul 7 '16 at 14:14
















How does this answer my question?
– user 170039
Jul 7 '16 at 13:56




How does this answer my question?
– user 170039
Jul 7 '16 at 13:56












I guess I should have said (which I have now inserted) that it is not necessary to have a topology before defining a basis. You can define a basis then generate a topology, or else you can find a topology for an existing basis.
– rschwieb
Jul 7 '16 at 13:59




I guess I should have said (which I have now inserted) that it is not necessary to have a topology before defining a basis. You can define a basis then generate a topology, or else you can find a topology for an existing basis.
– rschwieb
Jul 7 '16 at 13:59












So by "basis for a topology.." he means "basis for some topology (yet to be determined)", am I right?
– user 170039
Jul 7 '16 at 14:04




So by "basis for a topology.." he means "basis for some topology (yet to be determined)", am I right?
– user 170039
Jul 7 '16 at 14:04












@user170039 It can be: if you define a basis according to these axioms, you can generate a topology. But sometimes it goes the other way where you have a topology and you judiciously choose a basis for it to simplify your proofs.
– rschwieb
Jul 7 '16 at 14:08






@user170039 It can be: if you define a basis according to these axioms, you can generate a topology. But sometimes it goes the other way where you have a topology and you judiciously choose a basis for it to simplify your proofs.
– rschwieb
Jul 7 '16 at 14:08














The problem is that suppose we have a topology on $X$, say $mathcal{T}$. Then from the definition, it doesn't follow that a basis for $mathcal{T}$, say $mathcal{B}$ is a subset of $mathcal{T}$. To be precise, let $X=mathbb{R}$ with the standard topology. Then observe that both the set $mathcal{B}_{1}:={(a,b):a,binmathbb{R}}$ and $mathcal{B}_{1}:={[a,b]:a,binmathbb{R}}$ can be considered as a basis for the usual topology on $mathbb{R}$ according to the definition.
– user 170039
Jul 7 '16 at 14:14




The problem is that suppose we have a topology on $X$, say $mathcal{T}$. Then from the definition, it doesn't follow that a basis for $mathcal{T}$, say $mathcal{B}$ is a subset of $mathcal{T}$. To be precise, let $X=mathbb{R}$ with the standard topology. Then observe that both the set $mathcal{B}_{1}:={(a,b):a,binmathbb{R}}$ and $mathcal{B}_{1}:={[a,b]:a,binmathbb{R}}$ can be considered as a basis for the usual topology on $mathbb{R}$ according to the definition.
– user 170039
Jul 7 '16 at 14:14










up vote
2
down vote













A bit further in the book he will discuss how such a basis on a set generates a basis for a topology on that set by taking unions of basis elements.





Conversely, if you start with a space $X$ which is already topologized, then a collection $mathcal{B}$ of open subsets such that any open subset of $X$ can be written as a union of elements of $mathcal{B}$ is a basis for the set $X$ in the sense of the definition above. (And, via this basis, we recover the topology on $X$.)



Proof. We check the first condition. Let $x in X$. Since $X$ is open as a subset of itself, it may be written as a union of basis elements, $X = bigcup_mathcal{B} B$. Therefore by the definition of union, $x$ belongs to some $B in mathcal{B}$.



Now for the second condition. If $x$ belongs to the two open sets $B_1,B_2 in mathcal{B}$, it must also belong to the open set $B_1 cap B_2$. Writing this set as a union of basis elements, $B_1 cap B_2 = bigcup_mathcal{B} B$, we see again that by the definition of union there must exist some basis element $B_3 subset B_1 cap B_2$ containing $x$.






share|cite|improve this answer























  • How does this answer my question?
    – user 170039
    Jul 7 '16 at 13:56










  • @user170039 Is your question "What role does the topology on $X$ play in the definition?" If so, my answer is: your question is meaningless because $X$ isn't endowed with a topology.
    – Alex Provost
    Jul 7 '16 at 13:58












  • @user170039 The point is that you start with a set $X$ (with no topology) and use the basis to construct a topology on $X$.
    – Alex Provost
    Jul 7 '16 at 13:59












  • How does the converse part follows from the definition? Suppose $X$ is a set endowed with topology $mathcal{T}$. In the definition it is only mentioned that, "a basis for a topology on $X$ is a collection $mathcal{B}$ of subsets of $X$..", not as you wrote, "a collection of open subsets".
    – user 170039
    Jul 7 '16 at 14:10










  • @user170039 I wrote a proof of the converse. A collection of open subsets is, in particular, a collection of subsets...
    – Alex Provost
    Jul 7 '16 at 14:24















up vote
2
down vote













A bit further in the book he will discuss how such a basis on a set generates a basis for a topology on that set by taking unions of basis elements.





Conversely, if you start with a space $X$ which is already topologized, then a collection $mathcal{B}$ of open subsets such that any open subset of $X$ can be written as a union of elements of $mathcal{B}$ is a basis for the set $X$ in the sense of the definition above. (And, via this basis, we recover the topology on $X$.)



Proof. We check the first condition. Let $x in X$. Since $X$ is open as a subset of itself, it may be written as a union of basis elements, $X = bigcup_mathcal{B} B$. Therefore by the definition of union, $x$ belongs to some $B in mathcal{B}$.



Now for the second condition. If $x$ belongs to the two open sets $B_1,B_2 in mathcal{B}$, it must also belong to the open set $B_1 cap B_2$. Writing this set as a union of basis elements, $B_1 cap B_2 = bigcup_mathcal{B} B$, we see again that by the definition of union there must exist some basis element $B_3 subset B_1 cap B_2$ containing $x$.






share|cite|improve this answer























  • How does this answer my question?
    – user 170039
    Jul 7 '16 at 13:56










  • @user170039 Is your question "What role does the topology on $X$ play in the definition?" If so, my answer is: your question is meaningless because $X$ isn't endowed with a topology.
    – Alex Provost
    Jul 7 '16 at 13:58












  • @user170039 The point is that you start with a set $X$ (with no topology) and use the basis to construct a topology on $X$.
    – Alex Provost
    Jul 7 '16 at 13:59












  • How does the converse part follows from the definition? Suppose $X$ is a set endowed with topology $mathcal{T}$. In the definition it is only mentioned that, "a basis for a topology on $X$ is a collection $mathcal{B}$ of subsets of $X$..", not as you wrote, "a collection of open subsets".
    – user 170039
    Jul 7 '16 at 14:10










  • @user170039 I wrote a proof of the converse. A collection of open subsets is, in particular, a collection of subsets...
    – Alex Provost
    Jul 7 '16 at 14:24













up vote
2
down vote










up vote
2
down vote









A bit further in the book he will discuss how such a basis on a set generates a basis for a topology on that set by taking unions of basis elements.





Conversely, if you start with a space $X$ which is already topologized, then a collection $mathcal{B}$ of open subsets such that any open subset of $X$ can be written as a union of elements of $mathcal{B}$ is a basis for the set $X$ in the sense of the definition above. (And, via this basis, we recover the topology on $X$.)



Proof. We check the first condition. Let $x in X$. Since $X$ is open as a subset of itself, it may be written as a union of basis elements, $X = bigcup_mathcal{B} B$. Therefore by the definition of union, $x$ belongs to some $B in mathcal{B}$.



Now for the second condition. If $x$ belongs to the two open sets $B_1,B_2 in mathcal{B}$, it must also belong to the open set $B_1 cap B_2$. Writing this set as a union of basis elements, $B_1 cap B_2 = bigcup_mathcal{B} B$, we see again that by the definition of union there must exist some basis element $B_3 subset B_1 cap B_2$ containing $x$.






share|cite|improve this answer














A bit further in the book he will discuss how such a basis on a set generates a basis for a topology on that set by taking unions of basis elements.





Conversely, if you start with a space $X$ which is already topologized, then a collection $mathcal{B}$ of open subsets such that any open subset of $X$ can be written as a union of elements of $mathcal{B}$ is a basis for the set $X$ in the sense of the definition above. (And, via this basis, we recover the topology on $X$.)



Proof. We check the first condition. Let $x in X$. Since $X$ is open as a subset of itself, it may be written as a union of basis elements, $X = bigcup_mathcal{B} B$. Therefore by the definition of union, $x$ belongs to some $B in mathcal{B}$.



Now for the second condition. If $x$ belongs to the two open sets $B_1,B_2 in mathcal{B}$, it must also belong to the open set $B_1 cap B_2$. Writing this set as a union of basis elements, $B_1 cap B_2 = bigcup_mathcal{B} B$, we see again that by the definition of union there must exist some basis element $B_3 subset B_1 cap B_2$ containing $x$.







share|cite|improve this answer














share|cite|improve this answer



share|cite|improve this answer








edited Jul 7 '16 at 14:45

























answered Jul 7 '16 at 13:55









Alex Provost

15.1k22250




15.1k22250












  • How does this answer my question?
    – user 170039
    Jul 7 '16 at 13:56










  • @user170039 Is your question "What role does the topology on $X$ play in the definition?" If so, my answer is: your question is meaningless because $X$ isn't endowed with a topology.
    – Alex Provost
    Jul 7 '16 at 13:58












  • @user170039 The point is that you start with a set $X$ (with no topology) and use the basis to construct a topology on $X$.
    – Alex Provost
    Jul 7 '16 at 13:59












  • How does the converse part follows from the definition? Suppose $X$ is a set endowed with topology $mathcal{T}$. In the definition it is only mentioned that, "a basis for a topology on $X$ is a collection $mathcal{B}$ of subsets of $X$..", not as you wrote, "a collection of open subsets".
    – user 170039
    Jul 7 '16 at 14:10










  • @user170039 I wrote a proof of the converse. A collection of open subsets is, in particular, a collection of subsets...
    – Alex Provost
    Jul 7 '16 at 14:24


















  • How does this answer my question?
    – user 170039
    Jul 7 '16 at 13:56










  • @user170039 Is your question "What role does the topology on $X$ play in the definition?" If so, my answer is: your question is meaningless because $X$ isn't endowed with a topology.
    – Alex Provost
    Jul 7 '16 at 13:58












  • @user170039 The point is that you start with a set $X$ (with no topology) and use the basis to construct a topology on $X$.
    – Alex Provost
    Jul 7 '16 at 13:59












  • How does the converse part follows from the definition? Suppose $X$ is a set endowed with topology $mathcal{T}$. In the definition it is only mentioned that, "a basis for a topology on $X$ is a collection $mathcal{B}$ of subsets of $X$..", not as you wrote, "a collection of open subsets".
    – user 170039
    Jul 7 '16 at 14:10










  • @user170039 I wrote a proof of the converse. A collection of open subsets is, in particular, a collection of subsets...
    – Alex Provost
    Jul 7 '16 at 14:24
















How does this answer my question?
– user 170039
Jul 7 '16 at 13:56




How does this answer my question?
– user 170039
Jul 7 '16 at 13:56












@user170039 Is your question "What role does the topology on $X$ play in the definition?" If so, my answer is: your question is meaningless because $X$ isn't endowed with a topology.
– Alex Provost
Jul 7 '16 at 13:58






@user170039 Is your question "What role does the topology on $X$ play in the definition?" If so, my answer is: your question is meaningless because $X$ isn't endowed with a topology.
– Alex Provost
Jul 7 '16 at 13:58














@user170039 The point is that you start with a set $X$ (with no topology) and use the basis to construct a topology on $X$.
– Alex Provost
Jul 7 '16 at 13:59






@user170039 The point is that you start with a set $X$ (with no topology) and use the basis to construct a topology on $X$.
– Alex Provost
Jul 7 '16 at 13:59














How does the converse part follows from the definition? Suppose $X$ is a set endowed with topology $mathcal{T}$. In the definition it is only mentioned that, "a basis for a topology on $X$ is a collection $mathcal{B}$ of subsets of $X$..", not as you wrote, "a collection of open subsets".
– user 170039
Jul 7 '16 at 14:10




How does the converse part follows from the definition? Suppose $X$ is a set endowed with topology $mathcal{T}$. In the definition it is only mentioned that, "a basis for a topology on $X$ is a collection $mathcal{B}$ of subsets of $X$..", not as you wrote, "a collection of open subsets".
– user 170039
Jul 7 '16 at 14:10












@user170039 I wrote a proof of the converse. A collection of open subsets is, in particular, a collection of subsets...
– Alex Provost
Jul 7 '16 at 14:24




@user170039 I wrote a proof of the converse. A collection of open subsets is, in particular, a collection of subsets...
– Alex Provost
Jul 7 '16 at 14:24


















 

draft saved


draft discarded



















































 


draft saved


draft discarded














StackExchange.ready(
function () {
StackExchange.openid.initPostLogin('.new-post-login', 'https%3a%2f%2fmath.stackexchange.com%2fquestions%2f1852072%2fconfusion-regarding-munkress-definition-of-basis-for-a-topology%23new-answer', 'question_page');
}
);

Post as a guest















Required, but never shown





















































Required, but never shown














Required, but never shown












Required, but never shown







Required, but never shown

































Required, but never shown














Required, but never shown












Required, but never shown







Required, but never shown







Popular posts from this blog

Biblatex bibliography style without URLs when DOI exists (in Overleaf with Zotero bibliography)

ComboBox Display Member on multiple fields

Is it possible to collect Nectar points via Trainline?